You are on page 1of 114

The.Cleveland.Clinic.Cardiology.Board.

Review
The Dyslipidemias
QUESTIONS 1. Casey S. is an obese 54-year-old man (BMI = 35) with a history significant for obstructive sleep apnea and multiple joint complaints. A lower-extremity arterial duplex study, performed to work up the possibility of claudication, reveals noncritical peripheral arterial disease, with bilateral ankle-brachial indices (ABIs) of 0.8. He denies ongoing exertional chest discomfort or dyspnea, but lives a fairly sedentary lifestyle. Physical exam reveals an obese man with normal blood pressure (BP), an unremarkable cardiac exam, and 1+ dorsalis pedis pulses. His fasting lipids are as follows: TC 240 mg/dL, TG 250 mg/dL, HDL-C 35 mg/dL, LDL-C (calculated) 155. Initial therapy should include: a. TLC only b. TLC plus a statin, with the goal of reducing LDL-C to < 130 mg/dL c. TLC plus niacin or a fibrate, with the goals of reducing LDL-C < 100 mg/dL and TG < 150 mg/dL d. TLC plus a statin, with the goal of reducing LDL-C to <100 mg/dL (<70 mg/dL optional) e. TLC plus statin, as well as niacin or a fibrate, with the goals of reducing LDL-C < 100 mg/dL and TG <150 mg/dL View Answer Answer is d: Patients with a CHD equivalent (including clinically evident peripheral arterial disease) have an LDL-C goal of < 100 mg/dL, with the optional goal of <70 mg/dL. 2. Donna O. is a 71-year-old retired executive whose father died at age 52 of a massive MI. She is very worried about her own risk of a heart attack. She watches her weight (BMI = 23), does not smoke, and keeps physically fit, walking 3 miles on a treadmill three or four times a week. She denies angina or dyspnea on exertion, claudication, or history of TIA symptoms. Her BP is 120/80. Her fasting lipids are as follows: TC 250 mg/dL, TG 120 mg/dL, HDL-C 42 mg/dL, LDL-C (calculated) 151 mg/dL. Her calculated Framingham 10-year event risk is 5%. Initial therapy should include: a. Nothing beyond her current lifestyle measures b. Weight loss to bring BMI <20. c. TLC plus a statin to reduce LDL-C to <130 mg/dL, or optionally < 100 mg/dL d. TLC to reduce LDL-C to < 130 mg/dL e. Statin and niacin to reduce LDL-C < 130 mg/dL and increase HDL-C >50 mg/dL View Answer Answer is d: The patient has two major risk factors: age (woman 55 years), and family history o f CHD, with a 10year CHD event risk of < 10%. In such moderate-risk persons, TLC alone should be initiated if LDL-C is 130 mg/dL, and TLC plus drug therapy should be started for LDL-C 160 mg/dL. Because her LDL-C is 130 mg/dL, TLC measures should be initiated.

3. If Donna O. (Question 2) were found instead to have untreated HTN (systolic BP of 160), with all other data the same, what would be the preferred initial treatment, in addition to BP control? Her calculated 10-year Framingham risk score is now 11%. a. Nothing beyond her current lifestyle measures b. Weight loss to bring BMI <20 c. TLC plus a statin to reduce LDL-C to < 130 mg/dL, or optionally < 100 mg/dL d. TLC to reduce LDL-C to < 130 mg/dL e. Statin and niacin to reduce LDL-C < 130 mg/dL and increase HDL-C >50 mg/dL View Answer Answer is c: Given her HTN, the patient now has three major risk factors and a 10-year risk of between 10% and 20%, placing her in the moderately high-risk category. By ATP III, her goal LDL-C is < 130 mg/dL, with an optional goal of < 100 mg/dL, per the 2004 updates. Since her LDL-C is 130 mg/dL, a statin should be started. 4. Richard D. is referred to you for lipid management. He denies any first-degree relatives with history of CHD, but reports that two uncles and a distant cousin have had heart attacks. He is currently asymptomatic. His BMI is 28. His physical exam reveals arcus cornea and xanthelasmas, but no xanthomas, and a BP of 150/80. His fasting lipid profile is as follows: TC 300 mg/dL, TG 430 mg/dL, HDL-C 50 mg/dL, LDL-C (direct) 200 mg/dL. Which primary dyslipidemia is this patient most likely to have? a. Polygenic hypercholesterolemia b. Heterozygous familial hypercholesterolemia c. Familial combined hyperlipidemia d. Hyperapobetalipoproteinemia e. Familial endogenous hypertriglyceridemia View Answer Answer is c: Familial combined hyperlipidemia is a common dyslipidemia (1:33 to 1:100 persons) characterized by complex inheritance. Xanthomas are rarely present (unlike in heterozygous FH), but xanthelasmas and arcus cornea can be seen. Affected individuals generally exhibit a TC of 250 to 350 mg/dL, LDL-C of 200 to 300 mg/dL, and TG > 140 mg/dL (two thirds of patients with FCH have TG of 200 to 500 mg/dL. Patients with polygenic hypercholesterolemia have a similar lipid profile, except they do not generally have elevated TG.

5. What should the initial therapy be for Richard D. (Question 4)? a. Statin b. Fibrate c. Statin and antihypertensive agent d. Niacin e. Apheresis

View Answer Answer is c: Because this patient's TG are <500, LDL-C reduction has first priority. A statin should be initiated, as well as an antihypertensive agent. 6. Which of the following statements is not correct? a. 4S, CARE, LIPID, and HPS all involved secondary prevention of CHD. b. Data from HPS, ASCOT-LLA, and PROVE-IT/TIMI-22 were influential in lowering the recommended treatment goals for LDL-C in the 2004 ATP III updates. c. WOSCOPS and AFCAPS/TexCAPS were both primary CHD prevention studies that showed significant clinical benefits for statin therapy, with similar percentage reductions in LDL-C. The main difference between these trials was that subjects in AFCAPS/TexCAPS had considerably lower baseline LDL-C levels than those in WOSCOPS. d. Early angiographic trials of lipid lowering showed significant reductions in coronary events, though they were not designed to show this. e. ASCOT-LLA showed reductions in nonfatal MI, CHD death and all-cause mortality when patients with average lipids and HTN were treated with atorvastatin 10 mg daily for an average of 3.3 years. View Answer Answer is e: Although ASCOT-LLA showed reductions in nonfatal MI and CHD death, coronary events or procedures, stroke, and chronic stable angina, it did not show a reduction in total mortality. 7. Gary P. is an obese, nonsmoking, gregarious 44-year-old talk show host with treated HTN and no family history of CHD. He has no personal history of known CHD. He has had elevated LDL-C in the past, and is taking atorvastatin 20 mg/day. His latest lipid panel is as follows: TC 220, HDL-C 40, direct LDL-C 120, and TG 450. His calculated 10-year risk of a CHD event is 5%. After recommending lifestyle modifications, what is your first goal of drug treatment? a. Increase the HDL-C b. Lower the LDL-C c. Lower the non-HDL-C d. Lower the TG e. Lower the TG and increase the HDL-C View Answer Answer is c: This man is currently at goal for his target LDL-C of < 130 mg/dL. Given the fact that his TG are in the range 200 to 499, the next priority is to lower his non-HDL-C from its current level of 180 mg/dL to < 160 mg/dL. 8. Which of the following would lower non-HDL-C? a. Increase the dose of atorvastatin b. Add a fibrate c. Add niacin d. Add ezetemibe

e. All of the above View Answer Answer is e: Any of the therapeutic interventions would lower the non-HDL-C (TC minus HDL-C). However, risks of combination therapy and lack of long-term clinical trials assessing add-on therapy need to be considered. Emphasis on TLC with an increase in the statin dose would be an appropriate first step. 9. If the patient's TG were 600 mg/dL, what would be your next step in lipid management? a. Increase the HDL-C b. Lower the LDL-C c. Lower the non-HDL-C d. Lower the TG e. Lower the TG and increase the HDL-C View Answer Answer is d: When TG are <500 mg/dL, the priority is to reduce TG to <500 mg/dL, to avoid pancreatitis. 10. What would you prescribe for the patient in Question 9? a. Raise the dose of atorvastatin to 40 mg/day b. Add a fibrate c. Add niacin d. Add ezetemibe e. Increase the statin dose and add a fibrate View Answer Answer is b: This change could be most effectively achieved by adding a fibrate to his regimen.

13 - Stable Angina: Risk Stratification, Medical Therapy, and Revascularization Strategies


QUESTIONS 1. An 85 year old hypertensive male referred to cardiology clinic with stable NYHA FC III angina for 3 months treated with aspirin, metoprolol succinate 150 mg daily, isosorbide mononitrate 120 mg daily, and simvastatin 40 mg daily. On exam, the heart rate is 57, the blood pressure is 98/60 mmHg, and the cardiopulmonary exam is unremarkable. Resting ECG is within normal limits. An exercise stress test is significant for 2 mm horizontal ST depression and exercise limiting chest discomfort at 6 METs. The Duke Treadmill Score for this patient is: a. -9, intermediate risk b. -9, high risk

c. -10, intermediate risk d. -14, intermediate risk e. -14, high risk View Answer Answer is e: This patient with exercise limiting angina has a DTS of -14 which predicts a high probability of severe angiographic coronary disease. The Duke treadmill score is calculated as follows: DTS: Exercise time (minutes based on the Bruce protocol) - (5 maximum ST segment deviation in mm) - (4 exercise angina [0 = none, 1 = non-limiting, and 2 = exercise limiting]). Patients are classified as low, moderate, or high risk according to their score: Low-risk +5, Moderate-risk from -10 to + 4, High-risk -11. These patients have 3%, 10%, and 35% 5-year mortality, respectively. Patients with high risk Duke Treadmill Scores frequently have left main or three-vessel disease that would benefit from revascularization, while low risk patients have an excellent prognosis that is unlikely to improve with further evaluation or revascularization. 2. The most appropriate next step would be: a. Increase nitrate dose b. Increase beta blocker dose c. Add calcium channel blocker d. Cardiac catheterization e. Echocardiography View Answer Answer is d: This patient is maximized on medical therapy and unlikely to derive significant benefit from titration of his current medications or addition of further anti-anginals. Echocardiography is not warranted at this time in the absence of an abnormal ECG or clinical signs of heart failure or valvular disease. In patients with ischemic chest pain refractory to maximized medical therapy cardiac catheterization would be indicated.

14 - Unstable Coronary Syndromes


1. All of the following are recommended mechanisms to decrease bleeding complications in the management of unstable coronary syndromes, except: a. Decreasing the maintenance dose of aspirin from 325 to 81 mg daily b. Stopping clopidogrel a minimum of 5 days prior to a major surgical procedure c. Stopping the routine use of heparin after percutaneous coronary intervention d. Withholding the loading dose of clopidogrel e. Reducing the dose of heparin for patients who are also on aspirin and a glycoprotein IIb/IIIa inhibitor View Answer

Answer is d: When clopidogrel is used for either invasively or conservatively managed unstable coronary syndromes, a loading dose of 300 to 600 mg should be used. Giving 75 mg of clopidogrel daily without a loading dose would require up to 7 days to reach full antiplatelet effect. All of the other listed strategies may be effective in reducing bleeding complications. 2. All of the following are high-risk features of the TIMI risk score for stratifying patients with unstable coronary syndromes, except: a. Elevated cardiac biomarkers b. Age > 65 years c. Tachycardia d. Ischemic electrocardiographic changes e. A known coronary stenosis of more than 50% View Answer Answer is c: Although tachycardia (and hypotension) have been identified through the PURSUIT trial to be markers of high risk, they are not part of the formal TIMI risk model. All the other variables are components of the TIMI risk score. 3. All of the following therapies are class I recommendations for conservatively treated patients with unstable coronary syndromes, except: a. ACE inhibitor therapy b. Nitrate therapy c. Aspirin therapy d. Beta-blocker therapy e. Clopidogrel therapy View Answer Answer is a: All of the listed therapies are class I recommendations for stabilized patients with unstable coronary syndromes except ACE inhibitors, which are a class IIa recommendation. ACE inhibitors are strengthened to a class I recommendation if patients have diabetes or left ventricular dysfunction. Clopidogrel is a class I recommendation for both invasive and conservatively managed patients with unstable coronary syndromes. 4. All of the following are class III recommendations in the treatment of unstable coronary syndromes, except: a. Use of fibrinolytic therapy for non-ST-elevation acute coronary syndromes b. Use of abciximab for conservatively managed high-risk patients who continue to have ischemic symptoms c. The use of a low-molecular-weight heparin instead of unfractionated heparin for conservatively managed unstable coronary syndromes d. Use of nitroglycerin within 24 hours of sildenafil (Viagra) e. Invasive therapy in low-risk patients who present with a chest pain syndrome View Answer

Answer is c: There may be a marginal benefit of low-molecular-weight heparin over unfractionated heparin for conservatively managed patients, and this strategy is a class IIa recommendation. Nitroglycerin should not be used within 24 hours from the last dose of sildenafil. Fibrinolytics should only be used for ST-elevation myocardial infarctions. Ideally, high-risk patients should be managed invasively, but for high-risk individuals who defer invasive therapy or who have extensive co-morbidities and continue to have ischemic symptoms, the use of a glycoprotein IIb/IIIa inhibitor is a class IIa recommdation. However, eptifibitide or tirofiban should be used in this setting, while abciximab should be used only during invasive management. 5. Which of the following are causes of secondary angina? a. An anemic patient from a gastrointestinal bleed b. A dialysis patient with an arterio-venous fistula c. A dyspneic patient with underlying emphysema d. a and c e. a, b, and c View Answer Answer is e: Anemia, anterior-venous shunting, and hypoxemia can all cause demand ischemia. Note that a leftarm arterio-venous fistula can produce shunting as well as subclavian steal in patients with a previous left internal mammary artery graft.

15 - Acute Myocardial Infarction


1. In the TIMI risk score model, the variable that has the strongest prediction for subsequent 30-day mortality is: a. Low body weight (i.e., <67 kg) b. Tachycardia c. Advanced age (i.e., >75 years) d. Killip class II-IV at presentation e. Left bundle branch block at presentation View Answer Answer is c: Advanced age (>75 years) predicts the worst outcome for 30-day mortality and receives 3 points in the risk model. The other variables receive 1 to 2 points each. Hypotension (i.e., systolic blood pressure <90 mmHg) at presentation is also a high-risk variable and receives 3 points in the risk model. 2. Which of the following is not included in the differential diagnosis for electrocardiographic ST elevations? a. ST-elevation myocardial infarction b. Left ventricular aneurysm c. Hypokalemia d. Pericarditis

e. Left ventricular hypertrophy View Answer Answer is c: Among the electrolyte abnormalities, hyperkalemia, not hypokalemia can cause ST elevations that mimic ST-elevation myocardial infarctions. 3. Risk factors for intracranial hemorrhage during administration of fibrinolytics include all of the following except: a. Uncontrolled hypertension b. Advanced age c. Female gender d. Preexisting coagulopathy e. Morbid obesity View Answer Answer is e: Low body weight, not morbid obesity, is a risk factor for intracranial hemorrhage. 4. All of the following are class III recommendations except: P.153

a. Performing revascularization of non-infarct-related arteries at the time of primary PCI b. Waiting for cardiac biomarkers to return before making the diagnosis of an ST-elevation myocardial infarction c. Administering fibrinolytics to asymptomatic patients more than 24 hours from the onset of chest pain d. The use of a low-molecular-weight heparin along with fibrinolytics in elderly patients (i.e., >75 years old) or those with renal insufficiency e. The use of an oral ACE-inhibitor within 24 hours of an anterior ST-elevation myocardial infarction View Answer Answer is e: The use of an oral ACE inhibitor is generally recommended early in the hospital course as long as the patient is hemodynamically stable (usually at least 6 hours after presentation). In contrast, the use of an intravenous ACE inhibitor during the first 24 hours is not recommended. Non-infarct-related coronaries should not be revascularized except in the setting of cardiogenic shock. Fibrinolytics are recommended for ST-elevation myocardial infarction within 12 hours from the onset of chest pain. Administering fibrinolytics 12 to 24 hours from the onset of chest pain is generally not recommended, however individuals with stuttering chest pain during this time period may still be eligible to receive fibrinolytics. In elderly individuals (i.e., >75 years old) and/or those with renal insufficiency who also receive fibrinolytics, the use of unfractionated heparin is preferred over lowmolecular- weight heparin. 5. The fibrinolytic agent associated with the least intracranial hemorrhage is a. Alteplase (tPA) b. Streptokinase

c. Reteplase (rPA) d. Tenecteplase (TNK-tPA) View Answer Answer is b: Among the various fibrinolytic agents, streptokinase does not necessitate the use of heparin. This may help to explain the smaller incidence of intracranial hemorrhage seen with this agent.

16 - Fibrinolytic Therapy for Acute Myocardial Infarction


1. A 54-year-old man with a history of hypertension presents with chest pain that started 2 hours ago and is diagnosed in the Emergency Department with anterior STEMI. His blood pressure is 205/100 mm Hg. He receives intravenous metoprolol and the blood pressure falls to 170/85 mm Hg. What is the most appropriate reperfusion strategy? a. Fibrinolytic therapy b. Angiography and PCI c. Medical therapy alone and PCI at a later date View Answer Answer is b: The patient's admission systolic blood pressure is quite elevated and represents a relative contraindication for fibrinolytic therapy. Therefore, primary PCI is the preferred reperfusion strategy in this case. 2. A 75-year-old woman presents to a community hospital (without invasive capabilities) with 6 hours of chest pain and is diagnosed with inferior STEMI. The anticipated transfer to a medical center with primary PCI capability will take 2 hours. What is the preferred reperfusion therapy? a. Fibrinolytic therapy at the presentation hospital b. Transfer for angiography and PCI c. Half-dose fibrinolysis and GP IIb/IIIa inhibition at the presentation hospital and transfer for rescue PCI, if it fails to produce reperfusion View Answer Answer is a: The transfer-related delay of 2 hours tips the balance in favor of fibrinolytic therapy, according to the 2004 ACC/AHA Guidelines. 3. A 48-year-old man with an anterior STEMI receives fibrinolytic therapy with reteplase. 30 minutes after the second bolus he continues to have chest pain and his ECG demonstrates mild (estimated 30%) resolution of the ST segment elevations. What is the next correct step in his management? a. Readministration of a different fibrinolytic agent b. Administration of GP IIB/IIIA inhibitor c. Immediate coronary angiography and rescue PCI

d. Symptomatic relief of angina and heart failure View Answer Answer is c: The patient has failed fibrinolytic therapy as evidenced by ongoing ischemic symptoms and insufficient ST resolution (<50%). Coronary angiography and consideration for rescue PCI are advisable. 4. An 80-year-old woman with chest discomfort for the last 15 hours and progressively worsening dyspnea over the last few hours presents to the Emergency Department and is diagnosed with anterior STEMI. What, if any, reperfusion measures should be undertaken? a. Fibrinolysis b. Immediate coronary angiography and PCI c. Symptomatic relief of angina and heart failure View Answer Answer is b: The benefit of fibrinolytic therapy after 15 hours of ischemic symptoms is unclear. Moreover, it sounds as if the patient's MI may be complicated by heart failure, and she is best managed with medical therapy. An invasive strategy should be considered if she develops cardiogenic shock. 5. A 77-year-old thin woman with a history of gastroesophageal reflux disease presents in the first hour of an acute inferior STEMI. The blood pressure is normal. Invasive facilities are not readily available. What is the preferred strategy for reperfusion therapy? a. Fibrinolytic therapy b. Transfer for immediate coronary angiography and PCI c. Half-dose fibrinolysis and GP IIb/IIIa inhibition d. Symptomatic relief of angina only View Answer Answer is a: Even though advanced age, female gender, and low body mass index (BMI) are all risk factors for bleeding complications, the patient will have a mortality benefit with fibrinolytic therapy and should receive it.

HOW TO PREPARE FOR THE BOARDS


Thoroughly review the indications and contraindications of administering fibrinolytic therapy. Also, familiarize yourself with available adjunctive pharmacotherapy and its indications. The continued evaluation of the patient who has just received fibrinolytic therapy is important in terms of both ischemic symptoms and possible bleeding complications. When taking the examination, keep in mind that administering fibrinolytic therapy for acute STEMI is appropriate when significant delays are expected with primary PCI.

17 - Percutaneous Coronary Intervention


1. A 73-year-old man underwent PCI after a non-ST-elevation myocardial infarction. He was pretreated with 325 mg of aspirin and 600 mg of clopidogrel, plus unfractionated heparin and weight-adjusted abciximab were

administered during the intervention. A drug-eluting stent is deployed into the mid-LAD. Following the procedure, which of the following statements is correct? a. Both heparin and abciximab infusions should be stopped. The arterial sheath should be removed when the ACT < 150 seconds, and 4 hours later an abciximab infusion should be restarted and continued for 12 hours. b. Heparin should be stopped, the abciximab infusion should be continued for 12 hours, the arterial sheath should be removed when the ACT is < 150 seconds, and 75 mg of clopidogrel should be continued for up to 1 year. c. The heparin should be stopped, the abciximab infusion should be continued until the time of sheath removal; the arterial sheath should be removed when the ACT is <150 seconds. d. Heparin infusion at 700 U/h should be started, the abciximab infusion should be continued for 12 hours, the arterial sheath should be removed when the ACT is <150 seconds, and 75 mg of clopidogrel should be continued for 4 weeks. View Answer Answer is b: Heparin should be stopped, the abciximab infusion should be continued for 12 hours, the arterial sheath should be removed when the ACT is less than < 150 seconds, and 75 mg of clopidogrel should be continued for up to 1 year. In the EPIC trial, prolonged sheath dwells and high heparin dose resulted in more major bleeding complications with abciximab than with placebo (14% versus 7%). However, reduction in heparin dose and early sheath removal resulted in fewer major bleeding complications with abciximab than with heparin alone in both the EPILOG (2.0% versus 3.1%) and the EPISTENT (1.5% versus 2.2% major bleeding) trials. Recently, concern has arisen regarding late thrombosis with drug-eluting stents. Theoretically, complete healing of the stent may take up to 2 years. Currently, continuing clopidogrel for 6 to 12 months is recommended. 2. A 62-year-old woman with crescendo angina has presented for elective PCI to a known focal stenosis in a moderate-sized first diagonal branch. She is not taking medications and has a severe allergy to shellfish (hives), and she also reports an allergy to aspirin although she cannot specify the reaction. Which of the following statements regarding patient management is correct? a. Administer 100 mg hydrocortisone IV, 50 mg diphenhydramine (Benadryl) IV, and 600 mg clopidogrel orally before the procedure, then proceed with the scheduled intervention. b. Administer 40 mg methylprednisolone IV, 50 mg diphenhydramine (Benadryl) IV, 600 mg clopidogrel orally, and standard-dose abciximab before the procedure, then proceed with the scheduled intervention. c. Administer 100 mg hydrocortisone IV, 50 mg diphenhydramine (Benadryl) IV, 325 mg aspirin, and 300 mg clopidogrel orally before the procedure, then proceed with the scheduled intervention. d. Administer 100 mg hydrocortisone IV, 50 mg diphenhydramine (Benadryl) IV, 300 mg clopidogrel orally, and standard-dose abciximab before the procedure, then proceed with the scheduled intervention. e. Consult an allergy specialist and postpone the intervention. View Answer Answer is e: Consult an allergy specialist and postpone the intervention. Manifestations of aspirin sensitivity such as exacerbations of respiratory tract disease and urticaria can occur in up to 10% and 0.2% of the general

population, respectively An urgent intervention could be undertaken with the pharmacologic therapy described in the second option. However, an elective intervention should be postponed, awaiting appropriate workup and desensitization. There is little or no experience with aspirin desensitization in patients with aspirin-induced anaphylaxis. Potent platelet inhibitors, such as ticlopidine and clopidogrel, have demonstrated value in unstable angina and may have value in patients with aspirin allergy; pretreatment for 2 to 4 days prior to intervention is recommended to achieve optimal platelet inhibition. Abciximab has been shown to reduce ischemic complications following high-risk (EPIC and CAPTURE trials) and elective coronary interventions (EPILOG trial) in patients receiving aspirin; its role in the aspirin-allergic patient has not been defined but is theoretically appealing. Investigational oral platelet receptor antagonists may be useful. 3. A 50-year-old man presents to the emergency room (ER) about 100 minutes after the onset of chest pain. The pain is substernal, radiates to the left arm, and is associated with vomiting and diaphoresis. On examination, the patient is tachycardic with a heart rate of 104, hypertensive with blood pressure of 150/88. An electrocardiogram showed 3 mm ST elevation in the anterior and lateral leads. Symptoms were not relieved with sublingual nitroglycerin or beta-blocker therapy. An IV nitroglycerin drip was titrated to control the patient's symptoms. The closest interventional cardiology center is 120 minutes away. The patient has asked about the therapeutic options at this point. What can you tell him about the various therapeutic modalities? a. Primary PCI is recommended over thrombolytics because PCI has short-term mortality benefit, reduced reinfarction risk, and reduced risk of stroke, regardless of duration of symptoms and time required to PTCA. b. Thrombolysis is recommended over PCI because door-to-balloon time is 90 minutes. c. Half-dose thrombolysis should be recommended; then the patient should be transferred for PCI. d. The patient should be referred for emergency CABG for better survival advantage. View Answer Answer is b: Current ACC/AHA guidelines for management of patients with ST-elevation MI are dependent on the duration of symptoms, mortality risk of the STEMI, the risk of bleeding, and the difference between time to PCI and time thrombolytics. PCI is recommended over thrombolysis when experienced operators restore blood flow in 90 minutes. The patient in this question is relatively young, with a large STEMI that h as a high mortality risk. He has been having symptoms for 80 minutes and requires 120 minutes or more to have perfusion established with PCI. He is hemodynamically stable. Delayed treatment affects outcome negatively, whether treatment is PCI or thrombolytics. Thrombolysis is highly effective in restoring coronary flow in STEMI, particularly when the patient has presented within 3 hours of symptoms. Data from the National Registry of MI showed increased mortality when PCI is delayed beyond 2 hours. Hence, this patient will likely have better outcome if he receives thrombolysis. 4. A 62-year-old woman, current smoker, diabetic, hypertensive and hyperlipidemic, presented to the ER with a 2-day history of intermittent chest pain. On examination, she was tachycardic with a heart rate of 101 beats/min, and hypertensive with a blood pressure of 160/100 mm Hg. The patient does not have any signs of heart failure. An electrocardiogram showed 2-mm depression in V2-V6. The patient was started on heparin

and given IV metoprolol for rate control, as well as nitroglycerin. In addition, she was started on an eptifibatide infusion. The patient remained asymptomatic until her coronary angiogram. The left heart catheterization showed 90% stenosis in the proximal third of the LAD, 80% stenosis in the proximal RCA, with 30% disease in the middle P.183

circumflex artery. The best plan of treatment for this patient is a. PCI to both the LAD and RCA now b. Referral for two-vessel CABG c. PCI to the LAD with staged intervention to the RCA d. Medical management with no PCI or CABG View Answer Answer is b: The patient is diabetic with multivessel coronary artery disease. CABG has been shown to provide a survival benefit over PTCA according to the BARI trial. However, in the stenting era, one could recommend PCI with stenting, because there is no survival benefit of CABG over stenting according to ERACI-II, ARTS, and SOS. The patient is more likely to require further revascularization and more antianginal medication. Hence the recommendation according to the pre-drug-eluting stents ACC/AHA guidelines is to refer this patient to CABG.

18 - Risk Stratification and Post-Myocardial Infarction Therapy


1. A 52-year-old man presented to the Emergency Department with an acute anterior wall myocardial infarction and received successful lytic therapy. Physical exam findings were notable for a systolic blood pressure of 90 mm Hg, a heart rate of 120 beats/min, and rales at both lung bases. The most important determinant of 30-day mortality in this patient is a. Age b. Infarct location c. Killip class d. Systolic blood pressure e. Heart rate View Answer Answer is a: An analysis of 41,021 patients with acute MI enrolled in GUSTO-I, a trial of lytic therapy, found that age was the most significant predictor of 30-day mortality in a multivariable analysis. In addition, anterior infarct location, higher Killip class, elevated heart rate, and lower systolic blood pressure were predictors, although they were not as significant as age. Together, these five characteristics included 90% of the prognostic information in the baseline clinical data (Lee et al., Circulation. 1995;91:1659-1668).

2. The patient in Question 1 had an uncomplicated in-hospital course. All of the following are acceptable riskfactor stratification strategies except: a. Assessment of LV function b. Predischarge cardiac catheterization c. Submaximal stress on days 4 to 6 d. Symptom-limited stress on days 10 to 14 View Answer Answer is e: The current ACC/AHA Guidelines for STEMI recommend assessment of LV function as part of a riskstratification algorithm. It is acceptable to proceed to cardiac catheterization, particularly in patients with EF <0.40 or with high-risk features. In patients who do not undergo cardiac catheterization, it is recommended that those with an interpretable ECG, and who can exercise, undergo exercise stress testing, either as a submaximal stress test on days 4 to 6 or a symptom-limited test on days 10 to 14. EP testing is not part of the recommended algorithm for risk stratification. 3. The patient in Question 1 has a brief episode of chest pain (less than 1 minute) with transient ST depression on the morning of his scheduled submaximal stress test. The pain was relieved with one sublingual nitroglycerin tablet. You should: a. Proceed with submaximal stress as planned b. Wait two or three additional days and proceed with stress testing if he remains asymptomatic c. Order echocardiography to see if there have been any additional wall motion abnormalities d. Schedule for coronary catheterization prior to discharge P.198

View Answer Answer is d: Recurrent ischemia after myocardial infarction is a high-risk predictor, and patients with recurrent ischemia should undergo cardiac catheterization and revascularization as indicated. 4. BF is a 48-year-old man who presents for a submaximal stress test prior to discharge after successful thrombolysis for an inferior wall myocardial infarction. His baseline ECG demonstrated a complete LBBB but was unchanged during stress testing. He achieved 5.5 METs and the stress test was stopped because of general fatigue. You are asked to review his stress test and decide to: a. Discharge the patient home and schedule him for a symptom-limited stress test in 10 to 14 days b. Schedule a symptom-limited stress test in 2 to 3 weeks c. Perform cardiac catheterization because of the low METs achieved d. Repeat the stress test with perfusion imaging secondary to baseline LBBB View Answer

Answer is d: Left bundle branch block precludes interpretation of a stress ECG and is a contraindication to exercise ECG testing, in the absence of nuclear perfusion imaging. 5. JT is a 65-year-old woman who developed cardiogenic shock 10 hours after presenting with an anterior wall myocardial infarction. The most appropriate management strategy is a. Administration of thrombolytics b. Watchful waiting after initiation of inotropic support and insertion of an intra-aortic balloon pump c. Coronary angiography with revascularization within 18 hours of shock onset View Answer Answer is d: Baseline ECG abnormalities can preclude ECG stress test interpretation. LVH with strain, WPW, ventricular pacemaker, and baseline ST depression fall into this category. However, the ECG in RBBB is interpretable, as ST segments are generally normal in this condition.

19 - Complications of Myocardial Infarction


QUESTIONS 1. All of the following should raise suspicion of subacute free wall rupture, except: a. Intermittent chest pain, hypotension, and electromechanical dissociation b. Agitation and apprehension c. Intermittent, nonspecific, ST-T-wave abnormalities d. Pericardial effusion and echodensities in the pericardium e. Nonsustained ventricular tachycardia View Answer Answer is e: A high index of suspicion is needed to diagnose subacute free wall rupture. Accurate and timely diagnosis provides valuable time for surgical treatment before acute rupture and pericardial tamponade lead to death. Intermittent chest pain, nausea, electromechanical dissociation, and hypotension along with dynamic ST-Twave changes and agitation can all be present in cases of subacute rupture. 2. Which of the following statements regarding ventricular septal rupture complicating acute myocardial infarction is true? a. It is usually seen in elderly, hypertensive patients with a history of multiple prior infarctions. b. It is more common in anterior than inferior infarctions. c. A 4/6 holosystolic murmur indicates a large defect. d. Either echocardiography or right heart catheterization can be used as the initial diagnostic tool. e. Surgery should be delayed several weeks until infarct healing occurs. View Answer

Answer is d: In most series, the frequency of ventricular septal rupture was equal in anterior and inferior myocardial infarctions. Although it is usually seen in elderly hypertensive patients, many times it occurs in the setting of a first myocardial infarction. The intensity of the murmur is usually inversely proportional to the size of the defect. Both echocardiography and right heart catheterization can be used as initial diagnostic modalities. Surgery should be performed emergently. 3. Which of the following statements regarding papillary muscle rupture complicating acute myocardial infarction is true? a. Papillary muscle rupture is most frequently seen in large, anterolateral infarctions. b. Patients should be referred for emergency catheterization and percutaneous intervention. c. A harsh holosystolic murmur and systolic thrill are very common. d. Despite pulmonary edema or shock, overall left ventricular systolic function may be normal. e. Right heart catheterization is the diagnostic modality of choice. View Answer Answer is d: Many times a relatively small myocardial infarction may be the culprit in papillary muscle rupture. Hyperdynamic left ventricular function in the setting of a relatively small myocardial infarction may appear puzzling in a patient with severe pulmonary edema and cardiogenic shock. Papillary muscle rupture is most frequently seen in inferior and posterior myocardial infarctions because of the single blood supply of the posteromedial papillary muscle. Although coronary angiography may be needed before surgery, definitive treatment requires emergency surgery, not percutaneous intervention. In many patients a systolic murmur may be audible, but the absence of a murmur does not rule out presence of papillary muscle rupture. A systolic thrill is very uncommon in papillary muscle rupture, as opposed to ventricular septal rupture. The diagnostic modality of choice is echocardiography. 4. Case: A 75-year-old woman with a history of hypertension and hyperlipidemia presents to the Emergency Department with dyspnea and fatigue. Thirty-six hours prior to presentation, she experienced substernal chest pressure that was severe, sudden in onset, and persisted for a few hours. She provides a his tory of exertional chest discomfort for the past 2 months.

Exam: Blood pressure 95/60, heart rate 110, respiratory rate 28, pulse oximetry 88%, room air temperature 37C Neck: Elevated neck veins Lungs: Bibasilar inspiratory crackles halfway up posterior thorax Heart: PMI displaced laterally, palpable thrill over the left, fourth intercostal space, tachycardic, regular, S1 and S2 normal, 3/6 holosystolic murmur heard best at left, lateral sternal border

Extremities: Trace edema, somewhat cool, 2+ distal pulses throughout ECG: sinus tachycardia, Q-waves and T-wave inversion in leads V1-V5.

CXR: Cardiomegaly, pulmonary edema Labs: CK 500 U/L, CKMB 50 ng/ml, troponin-T 12 ng/mL, creatinine 1.5 mg/dL

A. In addition to ordering oxygen therapy, lasix, aspirin, and nitroglycerin, what should be performed next? (1) Arterial blood gas (2) Left heart catheterization (3) Transthoracic echocardiogram (4) Placement of a right heart catheter (5) Cardiac CT scan View Answer Answer is (3): The clinical picture is consistent with an anterior myocardial infarction that occurred more than 24 hours ago and the patient now presents in Killip Class III heart failure. Although the CHF may very well be secondary to a large anterior MI, her exam creates concern of a mechanical complication. In addition to oxygen therapy and initial measures to treat her pulmonary edema, she should have an emergency TTE. An arterial blood gas is not essential in this situation because the clinical picture is clear and CO2 retention is not a concern at the moment. One can follow her oxygenation status by pulse oximetry. Although urgent left heart catheterization is indicated in patients with AMI complicated by CHF and CGS, this patient is several hours out of her acute event and the route of revascularization will be dictated by the presence or absence of a mechanical complication. A ventriculogram can diagnose a VSD as well, but there is time to obtain a transthoracic echocardiogram in the present situation. A right heart catheterization is also indicated in this situation, but this can be performed in the catheterization laboratory or the coronary intensive care unit and does not need to be done immediately. A cardiac CT has no place in this situation. B. The above study demonstrated that the patient had an apical VSD, an akinetic anterior wall, and RV dilation. What should be the next step? (1) Cardiothoracic surgery consultation (2) Left heart catheterization (3) Placement of an intra-aortic balloon pump (4) Placement of a right heart catheter View Answer Answer is (1): The next step should be a consultation to cardiothoracic surgery. Make the page, and while you are waiting for a response, make arrangements for the patient to go to the catheterization laboratory and subsequently the coronary intensive care unit until surgery can be performed. In the catheterization laboratory, in addition to obtaining the coronary anatomy, an IABP and RHC can be placed. Further medical management can occur in the coronary care unit while plans for surgery are being made.

20 - The Science of Hemodynamic Measurements 21 - Electrocardiographic Stress Testing


1. A 40-year-old asymptomatic man with no risk factors undergoes stress testing as part of an Executive Physical program. His resting ECG is normal and he is taking no medications. He has an exercise capacity of 14 METs (13.5 minutes on the Bruce protocol), no angina, a peak heart rate of 180, and 1 mm of down-sloping ST-segment depression noted in lead V5. His Duke treadmill score is a. 9 b. 8.5 c. 7.5 d. 3.5 e. 2 View Answer Answer is b: DTS = 13.5 minutes - 5 1 mm of ST depression - 4 0 angina = 8.5. 2. Given these test results, the next most appropriate step is a. No further cardiac testing b. A stress imaging study c. A repeat stress test in 1 year d. Coronary angiography View Answer Answer is a: A DTS 5.5 implies low risk of death ( 1% per year) and therefore no further testing is needed. 3. A 55-year-old woman presents with intermittent substernal chest pain that radiates to the left arm. The pain is not clearly exertional and is not clearly relieved with rest. There is no history of gastrointestinal problems; her symptoms are not related to meals or body position. Her resting ECG is normal and she is taking no medications. She is referred for an exercise test and is found to have ST-segment depression. Assuming that the true, unbiased sensitivity of exercise ST-segment changes is 45% and the specificity is 85%, the likelihood that she has at least one 50% coronary artery stenosis is a. 0.25 b. 0.50 c. 0.75 d. 0.80 e. 0.90 View Answer

Answer is c: This is the positive predictive value, where PPV = (Sens)(Prev)/[(Sens)(Prev) + (1 - Spec)(1 - Prev)].The patients has atypical angina, and given her age and gender therefore has an intermediate-risk (0.50) pretest likelihood. Substituting values, the PPV is 0.75. 4. A 60-year-old man with chronic obstructive pulmonary disease (COPD) (FEV1 1.25) and chronic ischemic cardiomyopathy (EF 30%) is referred for metabolic stress testing, which shows the following: peak Vo2 15 mL/kg/min, peak Vco2 18 mL/kg/min, Vo2 at anaerobic threshold 10 mL/kg/min, peak VE 45 L/min. Which of the following is true? a. The test was submaximal. b. The primary limitation to exercise is cardiac. c. The primary limitation to exercise is pulmonary. d. The patient should be referred for cardiac transplantation. e. It is not possible to differentiate cardiac from pulmonary limitations to exercise in this patient. View Answer Answer is c: The MVV is 40 1.25 = 50. Given his VE of 45, he used up 90% of his breathing reserve. 5. A 60-year-old man presents with exertional pressure-like chest discomfort that is relieved with rest and that often radiates to the left arm and jaw. His resting ECG is normal. He is taking no medications. Which of the following is true? a. The patient should be referred for coronary angiography. b. The patient should have an exercise test to determine whether obstructive coronary artery disease is present. c. The patient should be referred for an exercise imaging study. d. The patient should have an exercise test to determine his short- and long-term prognosis. e. The patient need not have any test; he should be started on a beta-blocker, aspirin, and a lipid-lowering agent and then followed. View Answer Answers is d: The patient has typical angina and a very high pretest likelihood of disease. Exercise testing is appropriate to assess prognosis. If he is found to be at low risk, medical management will be appropriate.

22 - Nuclear Stress Testing


1. A 65-year-old man with a 45-pack-year smoking history, hyperlipidemia, intermittent claudication, and hypertension has been experiencing shortness of breath with exertion for 3 months. He is referred for an adenosine cardiac SPECT study for symptom evaluation. His medications include metoprolol, lisinopril, aspirin, theophylline, and simvastatin. During the adenosine infusion, the patient does not report any chest pain, nor are there any ST-segment changes on the electrocardiogram. High-degree atrioventricular block develops 30 seconds following the start of the adenosine infusion, prompting the cardiology fellow attending the stress

test to stop the infusion. The SPECT perfusion images are interpreted as normal, with no regional ischemia. Despite continuation of medical therapy the patient's symptoms persist, and 6 weeks later he is referred for cardiac catheterization. At catheterization there is a proximal 75% right coronary artery stenosis, a 75% to 80% proximal left anterior descending artery stenosis, and a 70% to 75% stenosis of the proximal circumflex artery. Possible reasons for the absence of a reversible perfusion defect on the cardiac SPECT study include all of the following except: a. Ingestion of a chocolate bar 3 hours before the test was performed b. Right bundle branch block on the resting electrocardiogram c. Provocation of balanced ischemia by the adenosine stress d. Failure to withhold the patient's medications prior to the test e. Termination of the adenosine infusion at 30 seconds View Answer Answer is b: Appropriate patient preparation is crucial for successful nuclear stress imaging. Recent ingestion of chocolate and/or theophylline could have blunted the hyperemic effects of adenosine, resulting in a falsely negative study. A 30-second infusion of adenosine might not have delivered a sufficient amount of the drug to produce adequate myocardial hyperemia. Alternatively, in a patient with proximal stenoses of nearly equal severity in each of the major coronary vessels, balanced ischemia is also a consideration; in this situation, a regional disparity in myocardial perfusion on the stress images is not identified because the impairment in flow reserve is similar in each of the three vascular territories. Right bundle branch block itself would not be expected to cause a false negative perfusion study. 2. Which of the following individuals is likely to benefit most from nuclear stress imaging? a. A 25-year-old man with midline chest pain, which is tender to the touch and intermittently responsive to ibuprofen. b. A 30-year-old woman who gets chest discomfort after eating highly seasoned food, but who has no trouble when she plays tennis three times a week. c. A 39-year-old male smoker with shortness of breath on exertion and a mildly elevated LDL cholesterol level. His father died suddenly at age 45, and his 42-year-old brother recently had two stents placed in one of his coronary arteries. The resting electrocardiogram shows nonspecific ST-T-wave changes. d. A 76-year-old man, former smoker, with hypertension and recent inferior wall myocardial infarction treated by placing two stents in the right coronary artery. He was awakened by an episode of chest pain that lasted almost 20 minutes and that has not responded to sublingual nitroglycerin. e. A 55-year-old female with hypercholesterolemia, hypertension, frequent heartburn, and increasing shortness of breath on exertion. On echocardiography, there is moderate left ventricular hypertrophy, and aortic valvular calcification with an estimated aortic valve area of 0.69 cm2. View Answer

Answer is c: The 39-year-old smoker has several cardiac risk factors and is in an intermediate-risk category for an adverse cardiac event. This patient is the one most likely to benefit from diagnostic testing, for a positive stress perfusion study will put him into a high-risk category, whereas a negative stress perfusion study will stratify him into a low-risk patient population. The young man and woman in a and b have noncardiac chest pain; they are in a low-risk population and are unlikely to derive a benefit from stress myocardial perfusion imaging. The patient in d has known coronary artery disease and an unstable clinical picture following recent coronary stenting, and would more appropriately be referred directly for repeat coronary angiography. The patient in e has moderately severe aortic stenosis, and would more appropriately be referred for cardiac catheterization and coronary angiography. 3. A 58-year-old male executive is seen for left-sided chest pain. He has a history of bilateral thumb pain for which he took a cox-II inhibitor for 2 years before switching to naproxen. He works long hours and admits to fatigue and loss of libido. He has an elevated lipoprotein a (Lpa) level but an otherwise normal lipid profile. The hs-CRP level is normal and a cardiac SPECT study 3 years earlier was normal. He undergoes an exercise cardiac SPECT and exercises to 10 METs on the Bruce protocol, achieving 106% of his MPHR. With exercise he experiences fatigue but no angina. No ST-segment changes are noted with stress. The myocardial perfusion images from the exercise study are shown below.

P.250

Based on the results of this scan, his cardiac mortality over the next 3 years can be estimated as a. 1.5% b. 3% c. 5% d. 6% e. >9% View Answer

Answer is a: The myocardial perfusion images in this middle-aged man with atypical chest pain and two cardiac risk factors (male sex, elevated lipoprotein a level) are normal. The risk of a cardiac event over the next 3 years in this patient is 1.5% (0.5%/year), according to one study. 4. A 71-year-old man with a history of hyperthyroidism, hypertension, and remote pulmonary embolism is referred for treatment of new-onset atrial fibrillation. He underwent a rest rubidium/dipyridamole stress rubidium-82 perfusion study.

The myocardial perfusion images are most consistent with a. Inferior ischemia b. Apical and inferior ischemia c. Anterior wall ischemia d. Diaphragmatic attenuation e. Normal perfusion scan View Answer

Answer is b: Reversible perfusion defects are identified involving the apical and inferior myocardial regions. In PET imaging, transmission images are used to correct the emisson images for attenuation, thus attenuation by the diaphragm should not influence the tracer concentration in the inferior wall. 5. A 36-year-old woman presents to the Emergency Room with atypical chest pain. She smokes and there is a family history of coronary artery disease. The electrocardiogram shows a normal sinus rhythm with early repolarization. Cardiac enzymes are negative and the patient is referred for stress myocardial perfusion imaging. The patient undergoes treadmill exercise using the Bruce protocol. She is able to complete Stage 2 of the exercise protocol (7 METS), being limited by leg fatigure. She does not experience chest pain with exercise. She achieves 92% of her maximal age-predicted heart rate. During stress, the electrocardiogram shows a new left bundle branch block. Rest thallium-201 and stress Tc-99m tetrofosmin images were obtained:

The myocardial perfusion images demonstrate a. Normal study with breast attenuation b. A fixed septal perfusion defect c. A reversible septal perfusion defect, indicating disease in the left anterior descending coronary artery

d. A reversible septal perfusion defect, reflecting the development of left bundle branch block with exercise e. A reversible septal perfusion defect of uncertain etiology View Answer Answer is e: A reversible septal perfusion defect is identified on the myocardial perfusion images. The reversible defect could reflect either the onset of left bundle branch block with stress or obstructive coronary disease in the left anterior descending artery or both. Therefore, the findings are equivocal for coronary artery disease. The patient had CT coronary angiography following the nuclear imaging study, and this did not reveal any coronary lesions. 6. An 80-year-old man is referred for a second opinion regarding the need for cardiac surgery. He sustained an inferior myocardial infarction 12 years ago. Over the last 4 years he has had increasing shortness of breath, but no angina. Nine months ago he had an echocardiogram that showed mild calcific aortic stenosis, with a left P.251

ventricular ejection fraction of 60%. Three months prior to presentation he was hospitalized for congestive heart failure. Echocardiogram again showed mild aortic stenosis, with a LVEF of 25%. Cardiac catheterization confirmed mild aortic stenosis and on coronary angiography there was multivessel coronary artery disease. Rest and stress rubidium-82 perfusion images, and 18FDG metabolic PET images, were obtained:

The PET scan demonstrates a. A small inferior scar b. Extensive septal, anterior, apical, and lateral ischemia c. A small inferior scar, with extensive septal, anterior, apical, and lateral ischemia d. Extensive myocardial hibernation involving the septal, anterior, apical, and lateral regions, with a small inferior scar e. Normal regional perfusion and metabolism findings suggest nonischemic dilated cardiomyopathy View Answer Answer is c: Extensive reversible perfusion defects are noted in the septal, anterior, apical, and lateral regions, and there is a small scar in the inferior region that is best identified on the short-axis images. Because of the extensive ischemia, the patient was referred for coronary revascularization. 7. In a patient with ischemic cardiomyopathy, which of the following scintigraphic findings suggests that abnormal regional function is unlikely to improve if coronary revascularization is performed? a. A defect on rubidium-82 PET images with preserved uptake on 18F-fluorodeoxyglucose PET images in the same area.

b. A reversible stress-induced perfusion defect in the same region on rest thallium-201/stress Tc-99m sestamibi SPECT images. c. The region exhibits a Tc-99m sestamibi SPECT perfusion defect with a relative tracer concentration of 55% of maximal myocardial activity. Relative tracer activity on PET images with 18F-fluorodeoxyglucose is 95% of peak maximal myocardial activity. d. A resting thallium-201 perfusion defect in which the relative tracer concentration is 40% of peak myocardial uptake, and which then increases to 90% of peak myocardial uptake on images obtained following thallium-201 reinjection. e. A matching defect on 13N-ammonia and 18F-fluorodeoxyglucose PET images. View Answer Answer is e: A matching defect on 13N-ammonia and 18F-fluorodeoxyglucose PET images is indicative of myocardial scar, and there is little chance that the region will exhibit improved function if revascularization is performed. Regions with perfusion-metabolism mismatches, or hibernating myocardium, as exemplified by the findings in a and c, are likely to improve functionally if revascularization is performed. Dysfunctional regions with reversible perfusion defects, whether in response to stress (b) or on rest/reinjection thallium-201 images (d), are also likely to benefit functionally from coronary revascularization. 8. A 57-year-old man with a history of coronary artery disease, prior myocardial infarction, and remote coronary artery bypass surgery was referred for evaluation for coronary revascularization because of recurrent angina and heart failure symptoms. There was a history of hyperlipidemia, hypertension, and deep venous thrombosis. An AICD had been placed 2 years before because of ventricular arrhythmias. On echocardiography, there is global left ventricular systolic dysfunction, with an LVEF of 20%. Rest and stress rubidium-82 perfusion and 18F-fluorodeoxyglucose PET images were obtained:

P.252

Which of the following statements is true regarding the scintigraphic findings? a. Left ventricular dysfunction is probably due to a nonischemic cardiomyopathy. b. On contrast magnetic resonance imaging, pronounced late enhancement will probably be observed in the lateral wall. c. A reversible perfusion defect is identified in the anterior wall. d. Coronary revascularization would be unlikely to improve the patient's heart failure symptoms. e. On histopathologic examination, extensive myocardial fibrosis would be expected if a biopsy of the lateral wall of the left ventricle were obtained. View Answer

Answer is c: A reversible perfusion defect is identified in the anterior wall. On the rest rubidium-82 perfusion and
18

F-fluorodeoxyglucose images, there is an extensive perfusion-metabolism mismatch consistent with myocardial

hibernation involving the anterolateral and inferolateral walls, as well as a portion of the inferior wall. There is a small inferior scar. The findings indicate that the patient would benefit from coronary revascularization. Prior histopathologic studies indicate that there is minimal fibrosis in areas with hibernating tissue, and therefore extensive late enhancement on contrast magnetic resonance imaging would not be anticipated.

23 - Stress Echocardiography 24 - Aortic and Pulmonary Valve Disease


1. A 42-year-old man with hypertension, but no prior cardiac history, presents with increasing dyspnea on exertion. Physical exam reveals a heart rate of 75 beats/min and blood pressure of 175/67. The jugular venous pattern (JVP) is unremarkable. S1 is soft, S2 is normal, and there P.273

is an early systolic sound. There is a soft II/VI SEM at RUSB radiating to the neck, and a III/VI decrescendo, holodiastolic murmur near LLSB. There is also a low-pitched diastolic rumble heard at the apex. The PMI is laterally displaced. Carotid pulsations are brisk and have a rapid upstroke, immediately followed by a second systolic pulsation. Femoral pulses are normal, and are slightly delayed compared to the radial pulse. Which of the following findings would you not expect to see on transthoracic echocardiography? a. Fluttering of the anterior mitral leaflet on M-mode echocardiography b. Mitral stenosis c. Bicuspid aortic valve d. Dilated LV cavity e. Coarctation of the aorta View Answer Answer is b: This patient is fairly young and has hypertension with a wide pulse pressure. The cardiac exam suggests a diagnosis of bicuspid aortic valve (younger patient with aortic insufficiency and an ejection click) with significant AI. The holodiastolic murmur is characteristic of chronic AI, and the displaced PMI suggests longstanding disease that has dilated the left ventricle. Likewise, bounding carotids and a bisferiens pulse are classic findings of AI. BAV usually results from fusion of the right and left coronary cusp leaflets, which then causes a posteriorly directed AI jet. This jet frequently hits the anterior leaflet of the mitral valve, which is manifested on M-mode echocardiography as fluttering of the anterior mitral leaflet and on exam as an Austin-Flint murmur. A history of hypertension in a young patient with AI should prompt an evaluation for aortic coarctation, because up

to 20% of patients with BAV also have coarctation. On physical exam, the slightly weaker and delayed femoral pulsations suggest that coarctation might be present. 2. You see a 17-year-old male adolescent in clinic, who is referred to you for evaluation of a murmur. He is well developed and physically active. His heart rate is 62 beats/min, and his blood pressure is 110/70. His JVP is normal. The cardiac exam shows normal S1 and S2. There is no third heart sound. There is a III/VI systolic ejection murmur (SEM) at the right upper sternal border (RUSB), which radiates to the carotids, and a soft diastolic murmur along the left sternal border. With Valsalva, the murmur softens. The carotid pulses are slightly delayed. Which of the following diagnoses is most likely? a. Bicuspid aortic valve b. Supravalvular AS c. Subvalvular AS d. Hypertrophic cardiomyopathy View Answer Answer is c: This patient has typical exam findings of subvalvular stenosis. In practice, subvalvular stenosis can easily be mistaken for native-valve AS. In younger patients, bicuspid or unicuspid valves are the main differential diagnosesboth of which can have findings of AS and AI. However, the absence of any ejection sound argues against aortic valvular pathology. Hypertrophic cardiomyopathy should also be considered in a patient this age; the slight delay in the carotids and the failure of the murmur to augment with Valsalva make HCM less likely. 3. You see a 25-year-old woman in clinic for a murmur. She is mildly mentally retarded but is sociable and conversational. Her eyes are widely spaced and her ears are low set. Her neck is webbed, and you note that she is rather short. Her jugular venous pattern has a prominent A wave. She has a pectus excavatum deformity of her chest. Cardiac exam reveals a sternal lift and a III/VI SEM at the LUSB, radiating to the left neck, which decreases with inspiration. You cannot appreciate any clicks. Which of the following statements is not true regarding this woman's condition? a. The mode of transmission is autosomal dominant. b. The genetic defect is linked to elastin. c. This valvular abnormality is not easily treated with valvuloplasty. d. ASD is a commonly associated cardiac abnormality. View Answer Answer is b: This patient has Noonan syndrome, an autosomal dominant disease characterized by mild mental retardation, characteristic facial features, and a variety of cardiac abnormalitiesthe most common of which are pulmonary stenosis, peripheral pulmonary stenosis, ASDs, and hypertrophic cardiomyopathy. The physical exam findings are typical of PS with increased a wave, RV left, and a systolic ejection murmur. Unlike other cases of congenital PS, patients with Noonan syndrome tend to have dysplastic pulmonary leaflets that do not cause an ejection click. They also are frequently not amenable to balloon valvuloplasty. Mutations in the gene for elastin are associated with supravalvular AS.

4. A 75-year-old man with prior bypass surgery is referred to you for shortness of breath and heart failure symptoms. He has a past history of hypertension and chronic obstructive pulmonary disease (COPD). His FEV 1 is 1.6 L. He also complains of occasional exertional angina. A recent adenosine nuclear scan revealed a fixed defect in the inferior wall, but no reversible defects. On gated images, the ejection fraction was 25%. On physical exam, his heart rate is 80 beats/min and his blood pressure is 110/80. He appears fatigued and somewhat frail. His JVP is elevated to 10 cm. He has bibasilar rales on pulmonary exam. Cardiac exam shows a normal S1 and a paradoxically split S2. There is a harsh III/VI SEM P.274

at the LSB, which peaks very late in systole and radiates to the carotids. A II/VI HSM is appreciated at the apex that radiates to the axilla. Carotid pulsations are delayed. You order an echocardiogram that confirms the severe LV systolic dysfunction. His LV is mildly dilated. The entire inferior and basal posterior walls are akinetic and thinned. The LAD and LCx territory is hypokinetic and hypertrophied. The aortic valve is heavily calcified and has poor leaflet excursion. Peak and mean gradients across the aortic valve are 27 and 17 mm Hg, respectively. By continuity, the AVA is 0.8 cm 2. There is 2+ mitral regurgitation due to posterior leaflet restriction. What is your next step in this patient's management? a. Suggest left heart catheterization to pursue percutaneous balloon valvuloplasty. b. Refer for cardiac surgery for AVR and MV repair. c. Institute diuretic therapy and afterload reduction to treat his congestive heart failure (CHF). d. Order dobutamine stress echocardiography. View Answer Answer is d: This patient has low-gradient AS with moderately severe LV dysfunction. His chief complaint is consistent with AS, but could be secondary to CHF or COPD. His physical exam, with narrow pulse pressure, paradoxically split S2, and SEM radiating to the carotids, all suggest AS. Loss of A2 is also consistent with severe AS. The nuclear stress test argues against ischemia. A relatively small fixed defect on nuclear study and preserved wall thickness in the left coronary territory both suggest that the LV dysfunction may be out of proportion to CAD, and may be secondary to valvular disease. For patients with low-gradient AS, dobutamine echo can be very helpful in assessing true stenosis versus pseudo-stenosis. In this patient, we would expect dobutamine to result in an increased EF (contractile reserve), increased gradients across the valve, and a valve area that remained severe. If he has pseudo-stenosis and a cardiomyopathy unrelated to the valve disease, dobutamine will increase cardiac output, but will not result in significant increases in the transaortic gradient or AVA. Patients with LV dysfunction who have contractile reserve and severe AS should undergo AVR. This patient has no contraindications to surgery, and thus valvuloplasty should not be considered definitive treatment. 5. A 37-year-old woman with an active history of IV drug abuse presents to the Emergency Department with abrupt-onset shortness of breath. She is tachycardic to 110 beats/min and has a systolic blood pressure of 95

mm Hg. Her boyfriend reports that over the past 7 days she has been febrile and anorectic. He also adds that the patient was born with an abnormal aortic valve. Which of the following findings is inconsistent with acute AI? a. Diminished S1 on auscultation b. Diastolic MR on echocardiography c. A holodiastolic murmur heard at the left sternal border. d. Premature closure of the mitral valve on 2-D echocardiography View Answer Answer is c: Acute AI typically has a brief, early diastolic murmur. Rapid equilibration of aortic and LVED pressures causes termination of the murmur by mid-diastole. All of the remaining answers are typical of acute AI. A diminished S1 may be seen in acute or chronic AI.

25 - Mitral and Tricuspid Valve Disease


1. Mitral valve prolapse is associated with which of the following? a. Heart failure b. Atrial fibrillation c. Stroke d. Pulmonary hypertension e. None of the above P.292

View Answer Answer is e: In a study by Freed et al. (2) of 84 subjects with mitral valve prolapse and 3,407 patients with no mitral valve prolapse, there was no significant association between presence of mitral valve prolapse and congestive heart failure, atrial fibrillation, cerebrovascular disease, or syncope. 2. What is the effect of successful mitral regurgitation repair on left ventricular ejection fraction? a. Stays the same b. Goes up c. Goes down View Answer Answer is c: In a study by Leung et al. (13) of 139 patients with isolated mitral regurgitation and no evidence of coronary artery disease, of whom 74 underwent uncomplicated valve repair, mitral regurgitation repair was

associated with decreased left ventricular ejection fraction and end-diastolic volume. However, end-systolic volume was preserved. 3. What level of regurgitant orifice area (ROA) is the criterion for severe (4+) mitral regurgitation? a. >60 cc b. >0.4 cm2 c. >200 cc/s d. >400 mm2 e. >0.6 cm2 View Answer Answer is b: The following cut points have been established for severe mitral regurgitation when using proximal isovelocity surface area (PISA) to calculate regurgitant surface area (ROA) (7):

<0.19 cm2 is mild 0.2-0.29 cm2 is moderate 0.3-0.39 cm2 is moderately severe >0.4 cm2 is severe

4. What is the most common etiology of tricuspid regurgitation? a. Rheumatic disease b. Prolapse or flail c. Trauma d. Carcinoid e. Left-sided heart failure View Answer Answer is e: The most common etiology of tricuspid regurgitation is left sided heart disease.

26 - Infective Endocarditis
1. All the following are true statements except: a. By definition, early prosthetic valve endocarditis refers to the development of infection within 90 days of surgery. b. Aztreonam is an acceptable alternative for the treatment of HACEK endocarditis. c. Gram-negative bacilli are important causative agents in hospital-acquired endocarditis. P.305

d. Endocarditis with Streptococcus bovis should prompt a search for a colonic neoplasm. e. Surgery is rarely necessary in IE with Brucella organisms. View Answer Answer is a: Early-onset prosthetic valve endocarditis is usually attributed to pathogens from perioperative contamination (health care-associated), and therefore the cutoff is 90 days from implant of the valve. Aztreonam (gram-negative organism only) will not be active against HACEK organisms. Staphylococcus aureus is the most common health care-associated cause of infective endocarditis (not gram-negative organisms). Brucella IE almost always requires surgery for cure. 2. All the following are false statements except: a. Mitral valve prolapse is a risk factor for the development of IE. b. Mechanical valves are as equally at risk as are native valves for IE during the first 90 days following surgery. c. MSSA is a common causative agent of hospital-acquired endocarditis. d. In persons with Staphylococcus aureus bacteremia, hemodialysis dependence has been shown to be a risk factor for the development of IE. e. Rheumatic valvulitis is still the most common predisposing factor for IE in the elderly in developed nations. View Answer Answer is d: Staphylococcus aureus is recognized as the most common health care-associated pathogen causing infective endocarditis. Patients receiving hemodialysis with indwelling vascular catheters are at particular risk for S. aureus bacteremia and subsequent endocarditis (Fowler VG et al., JAMA. 2005;293:3012). Mitral valve prolapse without regurgitation is not a high-risk condition for IE. Methicillin-resistant S. aureus is more common than methicillin-susceptible S. aureus as a cause of nosocomial IE in the United States. Rheumatic valvulitis is not the most common predisposing factor for IE in the developed world. 3. All the following are correctly matched associations except: a. Brucellaabattoir workers b. Coxiellasheep farmers c. Pseudomonas IEintravenous drug users d. Fungal IEprolonged antibiotic exposure e. HACEK IEveterinarians View Answer Answer is e: HACEKorganisms are not particularly associated with veterinarians. Brucella is a zoonosis and associated with abattoir workers. Similarly, Coxiella burnetii (agent of Q fever) is associated with parturient sheep. Pseudomonas has been associated with injected drug users (contamination with processing), and nosocomial fungal IE can follow prolonged antibiotic use (risk factor for fungemia). 4. The cutoff MIC for a penicillin-susceptible Streptococcus viridans is a. 0.5 g/mL b. 0.05 g/mL

c. 0.01 g/mL d. 0.1 g/mL e. 0.1 g/mL View Answer Answer is e: The NCCLS cutoff for S. viridans penicillin susceptibility is 0.1 g/mL. 5. Regarding the complications of IE, all of the following are true except: a. Four-vessel cerebral angiography is the current gold standard for diagnosing intracranial mycotic aneurysms. b. Mitral valve infection is a more common cause of congestive heart failure in IE than aortic valve infection. c. The definitive treatment of a splenic abscess resulting from embolization in IE is splenectomy. d. Transthoracic echocardiogram is inferior to transesophageal echocardiogram in detecting paravalvular extension of IE. e. Extracranial mycotic aneurysms are more common in the visceral arteries than in the upper-extremity arteries. View Answer Answer is b: Mitral valve IE is not more often associated with complications of CHF than aortic valve IE. MRA is not as sensitive as four-vessel angiogram for detection of mycotic aneurysms.

27 - Prosthetic Valvular Disease


1. What is the recommended anticoagulation therapy for each of the following valves? a. Carbomedics aortic valve b. St. Jude mitral valve P.313

c. Mitral bioprosthetic valve d. Starr-Edwards aortic valve e. Aortic bioprosthetic valve f. Single tilting Bjork Shiley mitral valve View Answer Answer: Embolic event rates are higher for mitral valves than for aortic valves and therefore generally require higher anticoagulation therapy. Caged ball and single tilting valves also carry greater embolic risk than double tilting mechanical valves. Bioprosthetic valves generally carry the lowest risk of embolization, yet according to AMA guidelines, anticoagulation is still recommend in the first 3 months after placement, although this varies according to institution. Thus, a. INR of 2 to 3 for life of valve

b. INR of 2.5 to 3.5 for life of valve c. Anticoagulation therapy with INR of 2.5 to 3.5 for first 3 months, then aspirin therapy thereafter d. INR of 3.5 to 4.5 for life of valve e. Anticoagulation therapy with INR of 2 to 3 for first 3 months, then aspirin therapy thereafter f. INR of 3 to 4 for life of valve 2. What is the preferred valve choice in each of the following clinical situations? a. A 45-year-old man with aortic valve endocarditis and aortic root abscess in the presence of a bicuspid aortic valve b. A 30-year-old man with a nonrepairable aortic bicuspid valve in the setting of severe symptomatic aortic insufficiency c. A 68-year-old woman with chronic lymphocytic leukemia, who has rheumatic mitral stenosis that is not amenable to valvuloplasty d. A 30-year-old woman with a nonrepairable aortic bicuspid valve in the setting of severe symptomatic aortic insufficiency View Answer Answers: a. In the setting of infection and, in particular, aortic root abscess, an aortic homograft is generally considered the best choice to prevent subsequent immediate reinfection. The durability of homografts was once thought to be superior to that of bioprosthetics, but recent experience has demonstrated this not to be the case. Furthermore, the difficulty of reoperation in such a patient (coronary reimplantation and subsequent extensive calcification) should be taken into account. b. A mechanical aortic valve with the lowest thromboembolic risk, such as a St. Jude or Carbomedics, is preferred, given its durability and potential to prevent future reoperation. The risk of anticoagulation for the patient must also be taken into consideration. c. Given the co-morbidity of chronic lymphocytic leukemia, which carries a decreased life expectancy yet not imminent death, a bioprosthetic valve is probably a good option. Of course, there are surgical considerations to the placement of a bioprosthetic, such as the larger profile of the valve secondary to its struts. If possible, however, the risk of anticoagulation should be avoided, given the low potential for reoperation in this patient. Recent data with bovine pericardial bioprosthetic valves demonstrate a higher-than-expected durability of ~85% at 15 years, which may ultimately shift the threshold for bioprosthetic valves to patients in their 50s and 60s. d. It is critical in such a situation to discuss valve selection with the patient. A young woman who would still like to have children should be informed of the risk of anticoagulation during pregnancy. Of course, a bioprosthetic valve in a young patient also carries a high risk of reoperation within the next 5 to 10 years. Some women may prefer to have a bioprosthetic valve, bear children in a timely fashion, and then later undergo a more definitive mechanical valve reoperation, to avoid the risks of anticoagulation during pregnancy. A homograft is not a good

choice in this case, given that its durability is not much greater than that of a bioprosthetic, but the reoperation is significantly more difficult. 3. Which of the following statements regarding prosthetic valve thrombosis (PVT) is false? a. The annual incidence of PVT is ~0.5% to 1%. b. Incidence is highest for the tricuspid position, followed by the mitral position, then the aortic position. c. Valve replacement and dbridement is generally performed for left-sided prosthetic valve thrombosis unless the thrombus is small or the patient has prohibitive surgical risk. d. Surgery is generally considered the treatment of choice for right-sided PVT. e. Cinefluoroscopy is a good option to determine restriction in occluder mobility. View Answer Answer is d: Fibrinolytic therapy is considered the treatment of choice for right-sided PVT because the consequences of distal embolization are less severe than in a left-sided prosthesis. Streptokinase and urokinase are the most common agents, and the success rate is ~82%, with a 12% rate of thromboembolism and 5% incidence of major bleeding for right-sided PVT. 4. When assessing transvalvular gradients across a prosthetic valve, all of the following can lead to a false assessment of prosthetic valve stenosis except: a. Patient-prosthesis mismatch b. Anemia c. Sepsis d. Regurgitation e. Pressure-recovery phenomenon View Answer Answer is a: Patient-prosthesis mismatch implies a true physiologic stenosis that is a result of the placement of a relatively small prosthesis, typically in the aortic position, that leads to a reduction in cardiac output. All prosthetic valves have an inherent relative stenosis, but when an inappropriately small prosthesis is placed, a patient can be left with a true gradient that is similar to that prior to the operation. Anything that increases cardiac output, such as P.314

anemia, as in the postoperative period, or sepsis, will increase flow through the prosthesis and produce higherthan-normal transvalvular gradients. Similarly, increased regurgitation, such as mitral regurgitation, which is often shielded on surface echo by prosthetic valves, will increase flow across the valve and produce a picture of pseudostenosis. Pressure-recovery phenomenon describes a false elevation in gradients that is obtained by echocardiography, typically as a result of turbulent flow just above a mechanical valve, which dissipates in the ascending aorta.

5. A 30-year-old woman with a history of an aortic mechanical aortic valve is found to be pregnant. She is thought to be 3 weeks pregnant. The best course of action for managing her anticoagulation during the pregnancy is a. Terminate the pregnancy, given the risk of warfarin embryopathy. b. Stop coumadin, admit her to the hospital, place her on IV heparin to a PTT two times normal, and once the PTT goal is achieved at steady state, send her home with IV heparin for the duration of her pregnancy until shortly prior to delivery, when the heparin will be stopped. c. Stop coumadin and place her on lovenox for the duration of the pregnancy. d. Stop coumadin, place her lovenox for the first trimester, then restart coumadin for the second trimester, and continue coumadin until the middle of the third trimester. Restart lovenox at that point and stop 12 hours prior to delivery. e. Stop coumadin, admit her to the hospital for IV heparin, and then convert to SQ heparin to get a consistent PTT of two times the control. Restart coumadin in the second trimester and continue until the middle of the third trimester. Restart SQ heparin again at that point until just prior to delivery View Answer Answer is e: Warfarin therapy during pregnancy carries a risk to both the fetus and the mother. The risk of warfarin embryopathy is ~30% in the first trimester. In addition, the risk of miscarriage or stillbirth is ~40%; thus, warfarin is an absolute contraindication in the first trimester of pregnancy. During the second and third trimesters, the risk of coumadin is less related to the embryopathy than fetal hemorrhage or placental compromise. Therefore, the risks of coumadin during this time period, although still somewhat high, are much more expectable. Lovenox therapy has limited data, but recent data suggest that low-molecular-weight heparin may not provide optimal anticoagulation in a hypercoagulable pregnancy. Continuous IV heparin therapy is not a practical option.

28 - Pathophysiology of Congestive Heart Failure


1. Which of the following statements about heart failure is true? a. It is a clinical syndrome. b. It can be caused by any form of heart disease. c. It is diagnosed primarily by history and physical exam. d. All of the above View Answer Answer is d: Heart failure, like renal failure or anemia, is a clinical syndrome with a constellation of signs and symptoms. It has many possible etiologies, since virtually any form of heart disease can lead to heart failure. Patients must have signs and symptoms (i.e., a low ejection fraction does not equal heart failure) that usually consist of dyspnea and fatigue at rest or with exertion. There must be underlying cardiac structural and/or functional abnormalities. There is no laboratory test for heart failure (i.e., a history and physical exam are necessary), though a plasma BNP level may help facilitate the diagnosis in certain settings.

2. The principal features of heart failure include all of the following except: a. Activation of the RAAS and SNS b. Left ventricular remodeling c. The ability to mount a reflex tachycardia d. Downregulation of -adrenergic receptors View Answer Answer is c: Patients with heart failure have well-documented disturbances of the autonomic nervous system, and are unable to mount a reflex tachycardia in response to upright tilt, orthostasis, intense vasodilation, or other volumedepleting stimuli. In fact, the extent of this blunted sympathetic response is coupled to the severity of heart failure and is predictive of a poor prognosis. Similarly, patients with heart failure do not fully activate the parasympathetic arm of the autonomic nervous system in response to systemic pressor activity with phenylephrine (there is less vagal-induced slowing of the heart rate). Heart-rate variability is also blunted in patients with heart failure, and is also associated with a poor prognosis. 3. The inability to exercise properly in heart failure is due to all of the following except: a. Reduced ejection fraction b. Skeletal muscle atrophy c. Endothelial dysfunction in peripheral vessels d. Inability to increase stroke volume and heart rate View Answer Answer is a: There has been a very reproducible and consistently poor relationship noted between resting ejection fraction (EF) and exercise capacity (r = 0.20-25) in patients with chronic heart failure. This is likely because exercise capacity is limited in patients with chronic heart failure, not by abnormal central hemodynamics, but by peripheral factors such as deconditioning and atrophy of skeletal muscles, changes in skeletal muscle oxidative enzymes, redistribution of blood flow away from skeletal muscles to more vital organs, and endothelial dysfunction in the peripheral vasculature due to a relative deficiency of local nitric oxide synthesis in blood vessels. 4. Which of the following statements about the prognosis for heart failure is true? (Select the best answer.) a. It is fairly easy to predict in individual patients. b. It is commonly assessed by measuring peak VO2 during exercise. c. It is closely coupled to ejection fraction in individual patients. d. It is commonly estimated by measuring neurohormones. View Answer Answer is b: There are almost as many prognostic factors in heart failure as there are stars in the clear night sky. Many of them are related to each other, and their independent contributions to prognosis are difficult to measure. Determining how much exercise the patient can do is perhaps the closest factor we have to a true gold standard for estimating prognosis. For example, the VO2 max should be less than 14 Ml/kg/min for a patient

to be considered for heart transplantation. Preserved exercise tolerance is a very powerful predictor of a better prognosis in patients with chronic heart failure. 5. Which of the following characterizes heart failure? a. Downregulation of 1- and 2-receptors b. Downregulation primarily of 1-receptors, with little change in 2-receptors c. Downregulation of G proteins and 1-and 2-receptors d. Increase in myocardial norepinephrine stores e. Intact baroreceptor function View Answer Answer is b: In chronic heart failure it is primarily the 1-receptor that is downregulated. The density of cardiac 2receptors is much less than that of the 1-receptors, and the 2-receptors may be less important in modulating positive inotropy. In addition to relatively selective P.324

1-receptor downregulation that occurs in chronic heart failure, there is important uncoupling of the G-stimulating protein from the -receptors, leading to a reduction in positive inotropic state. 6. All of the following neurohormones are associated with vasoconstriction, cell growth, hypertrophy, and sodium retention except: a. Angiotensin-II b. Norepinephrine c. Brain natriuretic peptide d. Endothelin e. Arginine vasopressin View Answer Answer is c: Natriuretic peptides modulate sodium and water (volume) regulation, vasodilation, natriuresis, antifibroblast proliferation, and anticollagen deposition, and have antiremodeling activity. Their biologic activities are nearly opposite those of angiotensin-II, norepinephrine, endothelin, and arginine vasopressin.

29 - Medical Treatment of Heart Failure


1. A 49-year-old man with dilated cardiomyopathy is admitted to the coronary care unit (CCU) for heart failure exacerbation. On examination, his respiratory rate is 30, with distended neck veins and a prominent S3. In addition to aggressive diuresis, a decision was made to start nesiritide. P.338

After the infusion, you notice hemodynamic changes. Which of the following changes is not related to the effects of nesiritide? a. Decrease in heart rate b. Decrease in BP c. Reduction in pulmonary capillary wedge pressure (PCWP) d. No change in stroke volume index e. All of the above f. None of the above View Answer Answer is a: BNP increases HR. All of the others are hemodynamic effects of BNP. 2. You have a 40-year-old man with a LVEF of 20%, ready to be discharged after treatment for HF exacerbation. At discharge, you explain to him the benefits of lisinopril, simvastatin, aspirin, digitalis, and furosemide. Finally, you want to explain the benefit of spironolactone to him. What will you tell the patient? a. Spironolactone in addition to standard therapy (ACE inhibitor, diuretic) does not decrease mortality or morbidity b. Spironolactone in addition to standard therapy (ACE inhibitor, diuretic) only decreases rehospitalization; it does not improve NYHA functional class c. Spironolactone in addition to standard therapy (ACE inhibitor, diuretic) decreases mortality and rehospitalization d. Spironolactone only benefits patients who are not on standard therapy. View Answer Answer is c: Spironolactone in addition to standard therapy decreases mortality and rehospitalization. 3. An 80-year-old woman with hyperlipidemia, hypertension, and diabetes mellitus has been well on enalapril, aspirin, simvastatin, glipizide, and metformin. Her friend tells her that losartan is better than enalapril. She wants your opinion. How would you advise her? a. Because the patient has no history of CHF, there is no reason to change her medication b. Losartan showed no significant reduction in the composite of mortality and morbidity from cardiovascular causes in diabetic, hypertensive patients, so there is no need to switch medications. c. Losartan did not show any mortality benefit but decreased the risk of MI; so she should have her prescription changed. d. Losartan did show mortality benefitbut only in patients younger than 60 years. View Answer Answer is b: Losartan showed a reduction in first hospitalization for heart failure in the RENAAL study. Losartan showed no effect on the rate of death. The composite of morbidity and mortality from cardiovascular causes was similar in the two groups, although the rate of first hospitalization for heart failure was significantly lower with losartan (risk reduction, 32%; p = 0.005). The reduction in MI was not statistically significant.

4. According to the ACC/AHA Practice Guidelines for Chronic Heart Failure, which of the following statements is false? a. A patient with a history of hypertension who does not have structural heart disease is classified as Stage A. b. A critically important aspect of the new Staging Classification is the increased focus on patients at high risk for the development of heart failure (Stage A). c. An asymptomatic patient with a prior MI and an ejection fraction of 25% is considered Stage B. d. A patient with severe mitral regurgitation and an ejection fraction of 45%, who complains of dyspnea upon exertion, is considered Stage C. e. None of the above View Answer Answer is e: None of the statements is false. Stage A is underlying predisposing disease without clinical or structural evidence of heart failure. The new criteria focus on disease treatment to prevent the development of heart failure. The patient with prior MI and no symptoms fits Stage B. The patient with mitral regurgitation and abnormal ejection fraction with symptoms fits the description of Stage C.

30 - Heart Transplantation
1. A 28-year-old man underwent heart transplantation 5 years ago for a presumed postviral dilated cardiomyopathy. He is seen at an urgent care clinic with complaints of a nonproductive cough, sore throat, and low-grade fever. He is diagnosed with an upper respiratory tract infection and prescribed clarithromycin, 500 mg twice daily for 10 days. Two weeks later, after completing antibiotics, he returns to the Transplant Clinic with complaints of generalized fatigue, shortness of breath, and a persistent nonproductive cough. His current immunosuppressive regimen includes prednisone, 10 mg daily; mycophenolate mofetil, 750 mg twice daily; and tacrolimus, 2 mg twice daily.

Physical exam: Sick-appearing man. Blood pressure is 142/92 mm Hg, heart rate is 98 beats/min and regular. Oropharynx is clear. Chest with bibasilar crackles. His abdomen shows no organomegaly or ascites. There is 3+ peripheral edema.

ECG: normal sinus rhythm, biatrial enlargement, incomplete right bundle branch block. Laboratory studies: Hematocrit 36%; white cell count 4,100/L; BUN 60 mg/dL; creatinine 5.3 mg/dL (baseline 1.3); tacrolimus level 29 ng/mL; MMF 4.5 ng/mL. Which of the following is the most likely explanation of the patient's current condition?

a. Humoral rejection with hemodynamic compromise b. Interaction of tacrolimus with mycophenolate mofetil, causing cellular rejection and thus acute renal failure

c. Immunocompromised patient with a viral syndrome d. Interaction of tacrolimus with clarithromycin, resulting in acute renal failure e. Noncompliance View Answer Answer is d: The patient's clinical picture is most likely secondary to tacrolimus (Tac) toxicity resulting in acute renal failure. Clarithromycin inhibits the cytochrome P450 system, causing increased levels of calcineurin inhibitors (tacrolimus and cyclosporine). The target trough level of tacrolimus > 12 months posttransplant is 8 to 10 ng/mL. The patient should discontinue tacrolimus until target trough levels are obtained. This clinical scenario could represent a rejection episode; however with the given history, tacrolimus toxicity is most likely the culprit. Tacrolimus taken with mycophenolate mofetil (MMF) is a common immunosuppression regimen. There is no increased incidence of renal failure or episodes of cellular rejection with this particular regimen. When taking Tac with MMF, the MMF dose should be decreased to lessen the likelihood of developing MMF toxicity (myelosuppression). The patient is definitely compliant, secondary to elevated trough levels of immunosuppressants. 2. A 23-year-old woman is seen for routine monthly posttransplant follow-up. She underwent an orthotopic cardiac transplantation 9 months ago for familial dilated cardiomyopathy. She has returned to college and reports that she saw an internist 3 weeks ago and was taken off of diltiazem, 60 mg twice daily, secondary to severe ankle swelling. She reports that her ankle swelling has improved and she is able to walk an hour a day without becoming fatigued or dyspneic. Current medications include prednisone, 10 mg daily; mycophenolate mofetil, 500 mg twice daily; cyclosporine, 75 mg twice daily; and bactrim, 1 tablet daily.

Physical examination: Blood pressure 110/78 mm Hg; heart rate 95 beats/min; RR 18/min. Chest is clear, PMI nondisplaced, normal S1 and S2. Extremities: no edema. ECG: Normal sinus rhythm at 95, with nonspecific ST-T abnormalities. Laboratory: Hematocrit 37%; white blood cell count 8,400/L, platelets 220,000/L; BUN 30 mg/dL; creatinine 1.1 mg/dL; cyclosporine 45 ng/mL; MMF level 2.2 ng/mL. Right ventricular biopsy shows prominent lymphocytic infiltrate with associated areas of myocyte necrosis with ISHLT Grade 3A rejection.

Which of the following statements is false?

a. Cyclosporine dose should be increased to achieve adequate trough levels. P.350

b. This rejection episode is most likely secondary to the discontinuation of diltiazem. c. There is no need to treat this rejection episode; the patient feels great and is hemodynamically stable.

d. Patients should report all changes in medications to the Transplant Clinic. View Answer Answer is c: The patient's episode of rejection is most likely secondary to discontinuing diltiazem 3 weeks earlier. Diltiazem inhibits the cytochrome P450 system, causing increased levels of cyclosporine and therefore requiring lesser dosages to achieve trough levels. Once diltiazem was discontinued, the dose of cyclosporine should have been increased. Almost all rejection episodes Grade 3A and higher and all rejection episodes that show hemodynamically instability (regardless of grade) are treated with augmented immunosuppression. It is important for transplant recipients to communicate any changes in medications to the Transplant Clinic, so these complications can be avoided. 3. All of the following are risk factors for posttransplant mortality except: a. Diabetes with end-organ damage b. Reversible pulmonary hypertension c. Active smoking d. Left ventricular assist device <30 days e. Active infection View Answer Answer is b: Risk factors for posttransplant mortality include but are not limited to short-term ventricular assist device use (<30 days), irreversible pulmonary vascular resistance (PVR >4 Wood units), mechanical ventilation at the time of transplant, active infection, active smoking, diabetes particularly with end-organ damage, and hepatic or renal insufficiency. 4. What is the most sensitive tool in detecting cardiac allograft vasculopathy posttransplantation? a. Serial echocardiography b. Intracoronary vascular ultrasound c. Positron emission scanning d. RV endomyocardial biopsy e. Coronary angiogram View Answer Answer is b: Serial echocardiograms are important for following graft function posttransplant; however, they are not very sensitive for detecting early CAV. Periodic right ventricular biopsy is the standard method of surveillance for cellular rejection, but adds little to the diagnosis of CAV. Positron emission scans are used to detect ischemia, and scarred and hibernating myocardium; however, they suffer from poor sensitivity in detecting CAV. Studies have shown intracoronary ultrasound (IVUS) to be the most sensitive tool in detecting and following the progression of CAV, compared to coronary angiograms. 5. Which of the following is true regarding posttransplant lymphoproliferative disease (PTLD)? a. Treatment includes reduction in immunosuppression therapy. b. It occurs in approximately 90% of all cardiac transplant patients.

c. The highest-risk group for developing PTLD is recipients who are Epstein-Barr virus (EBV) seronegative who receive an EBV-seronegative heart. d. The lymphomas that arise in PTLD are usually T cell in origin. e. The tumor usually arises 1 month after transplantation, with the cervical lymph nodes being the most common site. View Answer Answer is a: Posttransplant lymphoproliferative disease (PTLD) is a unique polyclonal B-cell lymphoma that occurs in approximately 3.4% of all heart transplant recipients. Ninety percent of PTLDs are associated with EBV; with EBV D+/R- being a high-risk group for developing PTLD. These tumors usually arise 12 to 18 months following transplant, after a mononucleosis-like illness (fever, sore throat, myalgias, and lymphadenopathy) and commonly are located intra-abdominally. Treatment includes decreasing the level of immunosupression, surgical debulking, cytotoxic chemotherapy, and radiation therapy if indicated; however, the response rate of advanced disease to treatment is poor (<50%).

31 - Devices for Heart Failure


QUESTIONS 1. The patient most likely to benefit from an implantable pulsatile ventricular assist device as a bridge to cardiac transplantation is a. A patient found to be in full cardiac arrest in the coronary care unit after being admitted with an acute coronary syndrome. b. A New York Heart Association Class II outpatient with ischemic congestive cardiomyopathy and atrial fibrillation, a QRS-complex duration of 130 milliseconds, moderate symptoms, physical findings of congestion, and a blood pressure of 140/80 mm Hg despite treatment with an angiotensin-converting enzyme inhibitor, aldosterone antagonist, and an angiotensin-receptor antagonist. c. A middle-aged-man with idiopathic dilated cardiomyopathy status post-biventricular pacemaker/ICD insertion who is hypotensive in response to oral medication therapies and requires continuous dobutamine infusion to maintain reasonable renal function but who is having intermittent and still problematic episodes of ventricular tachycardia. d. A postoperative coronary artery bypass patient who suffers a sudden ischemic event and, though resuscitated, has steadily increasing inotropic requirements, with blood pressure falling to 50 mm Hg despite polypharmacy with inotropic agents and vasopressers. He has not awakened postsurgery, and his mental status is questionable. e. A 30-year-old woman who develops pulmonary edema 1 week after a normal delivery and is admitted to the coronary care unit with a blood pressure of 90/50, a heart rate of 110 beats/min, atrial fibrillation, and profound respiratory distress. Hepatic and renal function are normal.

View Answer Answer is c: Appropriate selection of patients for ventricular assist device insertion as a bridge to transplantation is best described for this patient. This gentleman appears to have had aggressive therapies, including insertion of a biventricular pacemaker/ICD unit, but he remains compromised with hypotension and is inotropic dependent. Ventricular arrhythmias are compromising his current status. Insertion of a ventricular assist device will likely allow withdrawal of the inotrope, restore adequate systemic perfusion, and often in these cases, improve the ventricular arrhythmias as well as other organ function. This would then allow for successful cardiac transplantation. The patient described in (a) is in a very difficult situation, and because of the uncertainties regarding central nervous system integrity after a full cardiac arrest, is not the best candidate for aggressive mechanical circulatory support intervention. Sometimes, in newly presenting patients who are young and otherwise healthy, a temporary percutaneous Biomedicus pump or Biomedicus pump with extracorporeal oxygenation is used to see if the patient will in fact awaken and improve. The patient in (b) appears to be a bit too well for heart transplantation, and many things can be done, including insertion of a cardiac resynchronization device and intensification of medical therapies, to lower his blood pressure and treat the heart failure syndrome more optimally. The patient described in (d) resembles, in some ways, the patient described in (a). Using a smaller, less sophisticated temporizing device may be more prudent than implantation of a more permanent, and large, complex P.360

pulsatile system. The patient described in (e) is likely a peripartum cardiomyopathy patient who, in all likelihood, will have a reasonable response to medication therapies and correction of her atrial fibrillation. She may require transient intubation, but it is unlikely a device will be required. 2. Clinical goals that should be considered when mechanical circulatory support devices are inserted include: a. Bridge to a more sophisticated bridge b. Bridge to clinical improvement c. Permanent implantation without the goal of heart transplant d. None of the above, as the only consideration is bridge to transplant e. All of the above View Answer Answer is e: No longer should mechanical circulatory support be considered simply as a bridge to heart transplant. Many options now exist. Mechanical circulatory support devices can be used as a bridge to more sophisticated device implantation. They can also be used as a bridge to clinical improvement that perhaps will allow bridging to transplantation, even bridging the patient to recovery, with device removal or use of the device as a permanent alternative to transplantation. 3. A contraindication to mechanical circulatory support device implantation is a. Systolic blood pressure of 80 mm Hg on dobutamine at 10 g/kg/min

b. A serum creatinine of 2.8 mL/dL c. A platelet count consistently of 75,000 d. Hepatic cirrhosis on liver biopsy, with normal liver enzymes and bilirubin e. Presence of insulin-requiring diabetes View Answer Answer is d: One of the more feared difficulties after circulatory support device implantation is hepatic failure, which can quickly lead to coagulopathy and systemic organ failure. Even when liver enzymes and bilirubin levels are normal, the finding of hepatic cirrhosis on liver biopsy generally renders a patient at excessive risk for device implantation. This is the sole absolute contraindication included in this question. 4. Risk factors for death post-circulatory support device implantation include: a. Lower platelet count b. Higher white blood cell count c. Female sex d. All of the above e. None of the above View Answer Answeris d: According to the International Society for Heart and Lung Transplantation Mechanical Circulatory Support Database, female sex contributes to significant risk post-circulatory support device implantation. Reasons for this are not entirely clear, but it is more likely for an advanced female heart failure patient to be allosensetized, which will often prevent bridging to successful transplantation. Women are generally of smaller build, rendering implantation of some of the larger devices more technically challenging, with more frequent pulmonary hypertension. 5. The most common complication after mechanical circulatory support device implantation is a. Neurologic dysfunction b. Atrial fibrillation c. Bleeding requiring transfusion d. Right ventricular dysfunction e. Systemic infection requiring antibiotics View Answer Answer is e: According to the International Society for Heart and Lung Transplantation Mechanical Circulatory Support Device Registry, systemic infection requiring parenteral antibiotic therapy is the most common complication after device insertion and rises in continuous risk fashion over time (Table 31-9).

32 - Hypertrophic Cardiomyopathy

QUESTIONS 1. All of the following increase the gradient in hypertrophic cardiomyopathy except: a. Valsalva maneuver b. Squatting c. Amyl nitrite d. Isoproterenol e. Standing View Answer Answer is b: The Valsalva maneuver decreases venous return and thus decreases ventricular volume, thus accentuating the SAM and thus increasing the gradient. Amyl nitrite causes peripheral vasodilation and tachycardia. Both of these factors cause the left ventricle to decrease in size and thus increase the gradient. Isoproterenol increases the contractility and thus decreases ventricular volume, which increases the gradient. Standing decreases venous return and decreases ventricular volume. Squatting increases the vascular resistance and venous return, thus increasing ventricular volume and reducing the SAM, which reduces the gradient. 2. Which of the following are symptoms of left ventricular outflow tract obstruction? a. Dyspnea on exertion b. Syncope c. Chest pain d. Sudden death e. All of the above View Answer Answer is e: All of the above are classic symptoms of left ventricular outflow tract obstruction. 3. All of the following are true of the Brockenbrough response except: a. There is increased filling of the left ventricle with the compensatory pause. b. The premature beat causes a decrease in contractility in HCM but not in normal individuals. c. There is an increase in ventricular pressure in both normal individuals and in patients with hypertrophic cardiomyopathy. d. The is a decrease in aortic pressure in HCM. e. There is an increase in aortic pressure in normal individuals. View Answer Answer is b: The Brockenbrough maneuver causes the contractility to increase in both normal individuals and in patients with HCM. All of the other statements are true. P.377

4. Echocardiography is the primary clinical modality for diagnosing hypertrophic cardiomyopathy. Which of the following findings is (are) commonly seen in HCM? a. A septum >15mm b. Preclosure of the aortic valve c. Anterior displacement of the papillary muscles d. Elongated mitral leaflets e. All of the above View Answer Answer is e: The definition for the diagnosis of HCM is that the septum must be 15 mm or greater in the absence of any disease know to cause hypertrophy. Preclosure of the aortic valve is commonly seen on a M-mode echo of the aortic valve in the presence of left ventricular outflow tract obstruction. Anterior displacement of the papillary muscles is frequently seen in HCM and contributes to the development of outflow tract obstruction. Elongated mitral leaflets have been recognized for some time in HCM but are now more easily seen with better instrumentation. 5. All of the following drugs are useful in the treatment of HCM except: a. Metoprolol b. Disopyramide c. Enalapril d. Diltiazem e. Phenylephrine View Answer Answer is c: Enalapril is an angiotensin-converting enzyme inhibitor, and it can worsen obstruction by causing peripheral vasodilation. Metoprolol is a beta-blocker and thus, by slowing the heart rate, may allow for prolonging diastolic filling and lessen the provocable outflow tract gradient. It is also a negative inotrope. It is somewhat less helpful if there is resting obstruction. Disopyramide has a negative inotropic effect on the left ventricle and thus frequently diminishes left ventricular outflow tract obstruction. Diltiazem is a calcium channel blocker that has some negative inotropic and may lessen left ventricular outflow tract obstruction. In addition, it improves diastolic filling. Phenylephrine may be life saving in the treatment of hypotension-associated severe left ventricular outflow tract obstruction. It is a pure vascular constrictor and does not increase the contractility of the heart. 6. Which of the following is a risk factor for sudden death in HCM? a. Septal thickness >30mm b. Prolonged or repetitive episodes of nonsustained ventricular tachycardia c. Family history of sudden death d. Syncope or near- syncope e. No change or a decrease in blood pressure with exercise f. All of the above

View Answer Answer is f: Although all these factors have been shown to have a high negative predictive accuracy, the positive predictive accuracy is low.

33 - Myocarditis and Dilated Cardiomyopathy


QUESTIONS A 45-year-old woman came to see you because of intermittent chest pain and progressive shortness of breath for the last 3 days. She was seen last week by her primary physician because of sinusitis and was placed on azithromycin for 5 days without relief. Exam: Blood pressure 110/80 mm Hg, pulse 110 beats/min regular, jugular venous pressure 10 cm H2O, bibasilar rales, S3 gallop, 1-2+ pedal edema Electrocardiogram: Sinus tachycardia, nonspecific T-wave changes Echocardiography: Left ventricular ejection = 25%, 1-2+ mitral regurgitation 1. Which of the following is not an appropriate next course of action? a. Cardiac catheterization b. Endomyocardial biopsy to rule out acute lymphocytic myocarditis c. Start diuretics and ACE inhibitors d. Blood testing for thyroid function tests View Answer Answer is b: New-onset heart failure in a relatively young woman who had a recent bout of upper respiratory tract infection can be a potential clinical presentation of acute myocarditis. That being said, the usual course of action should involve cardiac catheterization to rule out coronary ischemia, blood testing to rule out reversible causes of heart failure such as hypo- or hyperthyroidism, and commencing therapy with diuretics, ACE inhibitors, and betaadrenergic blockers. Routine endomyocardial biopsy, even though it may provide the definitive diagnosis, does not change the patient management, and should be reserved for when patients require further evaluation because of decompensation. The patient's blood work and autoimmune work-up were negative. Cardiac catheterization revealed normal coronary arteries. Her clinical course deteriorated rapidly over the course of the next few days, requiring hospital admission for decompensated heart failure. She was found to be in cardiogenic shock, requiring inotropic support for stabilization. An endomyocardial biopsy was performed, and the preliminary results suggested acute lymphocytic myocarditis according to the Dallas criteria. Her cardiac index and hemodynamics were stable other than frequent nonsustained ventricular tachyarrhythmia. P.386

2. Based on this information, what is the appropriate therapeutic intervention? a. Intravenous Solu-Medrol b. Implantable cardioverter-defibrillator (ICD) c. Plasmapheresis d. No additional therapy at this point View Answer Answer is d: The patient now presents with fulminant myocarditis, requiring inotropic support. She should remain on supportive therapy, and there is no supporting evidence to recommend immunosuppression therapy at this stage. It would also be inappropriate for her to receive an ICD in this acute setting. 3. What is her 5-year prognosis if she survives this acute event? a. 93% b. 80% c. 65% d. 45% View Answer Answer is a: McCarthy and colleagues (16) compared the long-term prognosis between fulminant and acute myocarditis, and found a 93% survival for those suffering from fulminant myocarditis at 1-year follow-up, which was maintained for the next 10 years. This is significantly better from those presenting with acute myocarditis (45% at 11 years). 4. Which of the following patients would have a worse prognosis? a. A 29-year-old man with giant-cell myocarditis b. A 27-year-old woman with fulminant myocarditis c. A 3-year-old man with idiopathic restrictive cardiomyopathy d. A 25-year-old woman with postpartum cardiomyopathy View Answer Answer is a: All the choices have poor prognosis except for fulminant myocarditis if supported, but giant-cell myocarditis has the worst prognosis, and cardiac transplantation should be considered.

34 - Pulmonary Hypertension
QUESTIONS 1. A 45-year-old woman with a history of hypertension presented with dyspnea upon exertion. You obtained an echocardiogram showing normal left ventricular systolic and diastolic function, normal right ventricular

size and function, normal valvular function, but an estimated RVSP of 56 mm Hg with 1-2+ tricuspid regurgitation. Your next step should be a. To perform a pulmonary angiogram b. To perform to right heart catheterization c. To start oral bosentan therapy and follow up in 6 weeks d. To repeat an echocardiogram in 6 months View Answer Answer is b: Echocardiographic estimations of right ventricular systolic pressures may potentially overestimate the true pulmonary artery pressures, and should be confirmed by right heart catheterization in the setting of a clinical suspicion for pulmonary hypertension. 2. Routine diagnostic work-up for PAH in a patient with CREST syndrome and presenting with dyspnea upon exertion should include all of the following except: a. Transthoracic echocardiography b. Right heart catheterization c. Pulmonary function testing with DLCO d. Genetic testing for BNPR2 mutation View Answer Answer is d: CREST syndrome patients have a higher probability of developing pulmonary hypertension, and should be screened, particularly with symptom onset. Echocardiography and right heart catheterization are reasonable tools to detect pulmonary hypertension, and pulmonary function testing may reveal lung parenchymal abnormalities. Genetic screening is indicated only in familial cases of pulmonary hypertension, and the incidence of BMPR2 gene mutation in patients with CREST syndrome and pulmonary hypertension is actually low. 3. Poor prognostic variables for PAH include all of the following except: a. Septal shifting b. Presence of pericardial effusion c. Dilated right atrium d. Walks 360 m in 6 minutes View Answer Answer is d: Septal shifting, presence of pericardial effusion, and dilated right atrium are all poor prognostic signs for elevated right ventricular preload and pulmonary hypertension. In clinical trials, a distance of less than 332 m in a 6-minute walk test is considered poor functional capacity with a poor prognosis.

35 - Diastolic Heart Failure

1. A 70-year-old man presents with dyspnea on exertion. Physical examination reveals a blood pressure of 160/95 mm Hg and pulmonary rales at the base. An echocardiogram reveals left ventricular hypertrophy and the following Doppler findings.

Which of the following medications would not be an initial choice? a. Digoxin b. Metoprolol c. Furosemide d. Ramipril View Answer Answer is a: Digoxin. The figure depicts restrictive filling (Stage 3 diastolic dysfunction) in a patient with left ventricular hypertrophy and therefore digoxin is not indicated. 2. Which parameters are relatively preload independent? a. Mitral inflow E wave b. Tissue Doppler echo annular E wave c. Color m-mode flow propagation velocity d. b and c View Answer Answer is d: Both tissue Doppler echocardiography annular E wave and color m-mode flow propagation are measures of P.402

relaxation and are relatively preload independent. Mitral inflow E wave is dependent on preload. 3. Which of the following mitral inflow patterns is associated with the worst prognosis? a.

b.

c.

View Answer Answer is c: Restrictive pattern. Restrictive physiology has been shown to be associated with the highest mortality. 4. Which of the following is the most common symptom associated with diastolic heart failure? a. Dyspnea at rest b. Exertional dyspnea c. Paroxysmal nocturnal dyspnea d. Chest pain View Answer Answer is b: Exertional dyspnea. With exertion, diastolic filling worsens and left ventricular filling pressure increases, resulting in dyspnea. 5. A 60-year-old obese woman is experiencing increasing dyspnea, fatigue, and leg swelling. Physical exam reveals a mildly distressed patient with visible dyspnea and 3+ bilateral leg edema. Auscultation is difficult and only distant heart sounds can be discerned. Electrocardiogram shows low voltage and a pseudoinfarct pattern. At this time, which is the best diagnostic study? a. CT scan

b. Angiography c. Echocardiogram with respirometry d. MRI View Answer Answer is c: Echocardiogram with respirometry. Echocardiography is the modality of choice for the initial assessment of cardiac amyloidosis.

36 - Chest Radiography: What the Cardiologist Needs to Know 37 - Cardiac MRI and CT
1. A 51-year-old man presents to your clinic for evaluation of progressive exertional dyspnea over the last 6 months. On physical examination, his heart rate is 85 beats/min, his respiratory rate is 22, and his blood pressure is 108/65 mm Hg. His jugular venous pulse is visible 8 cm above the sternal angle at 45 degrees. The PMI is sustained but normal in location. He has an S4 gallop and 1 + bilateral pedal edema. A PA and lateral chest x-ray are unremarkable. A transthoracic echocardiogram reveals normal left and right ventricular systolic function with mild left ventricular hypertrophy and abnormal diastolic function. A cardiac MRI with gadolinium contrast is obtained.

Still frame gradient echo image, four-chamber view.

Corresponding delayed hyperenhanced image, four-chamber view.

Based on these images, the next most appropriate clinical step is a. Endomyocardial biopsy b. Fat pad biopsy c. Initiate corticosteroid therapy d. Surgical pericardial stripping View Answer

Answer is b: Fat pad biopsy. Although the mildly thickened ventricular myocardium is consistent with several different etiologies of cardiomyopathy, the diffuse pattern of hyperenhancement throughout the left ventricle on delayed P.434

hyperenhanced black blood MRI images (second figure above) is typical of cardiac amyloidosis. Cardiac sarcoidosis, which might be an indication to begin corticosteroid or other immunosuppressive therapy, typically demonstrates patchy areas of hyperenhancement, along with ventricular wall thinning and aneurysms, most commonly along the basal anteroseptal wall. Cine gradient echo images in sarcoidosis may demonstrate normal or impaired left ventricular systolic function, often with regional wall motion abnormalities. Fat pad biopsy is often preferred over endomyocardial biopsy for evaluation of amyloidosis because of its less invasive nature and higher sensitivity. Although there may be a role for immunosuppressive therapy in specific subtypes of amyloidosis, histologic diagnosis should be confirmed before therapy is initiated. There is no thickening of the pericardium, conical deformity of the ventricles, or atrial enlargement on these images to suggest constrictive pericarditis, making pericardial stripping inappropriate. 2. You are asked to see a 58-year-old woman in the emergency room who has presented with intermittent retrosternal chest pain without radiation lasting for less than 1 minute and a single episode of rest pain lasting 10 minutes today. She states she has been having these symptoms since shoveling snow 1 week prior to presentation. Her past medical history is significant for gastroesophageal reflux, for which she takes an overthe-counter H2 blocker infrequently. She takes no other medications. She was told at a health screening fair a few months ago that her cholesterol levels were high, but she has not seen her family physician about it. Physical examination is unremarkable. ECG reveals normal sinus rhythm with no ischemic changes. Initial laboratory evaluation, including a portable chest x-ray and cardiac enzymes, are within normal limits. A cardiac CTA is obtained to further evaluate the etiology of her chest pain.

Curved multiplanar reformatted (MPR) image of the aortic valve ( black arrow) and right coronary artery (white arrow).

Based on this image, the next most appropriate step is a. Begin therapy with an angiotensin-converting enzyme (ACE) inhibitor, beta-blocker, and diuretic, b. Begin therapy with a proton pump inhibitor. c. Obtain a transthoracic echocardiogram to evaluate for aortic stenosis. d. Refer the patient for a nuclear stress study. e. Refer the patient for cardiac catheterization.

View Answer Answer is e: Refer the patient for cardiac catheterization. The curved MPR image reveals a noncalcified atherosclerotic plaque in the mid-RCA associated with severe luminal stenosis, which can be compared to her corresponding coronary angiogram (see image below). The low attenuation characteristics of this lesion on coronary CTA suggest that it is a noncalcified plaque, unlike the higher-attenuation calcified plaque that occurs more proximally. Current CT technology does not allow precise quantification of coronary stenoses as is done with invasive angiography. Therefore, most lesions are graded as mild (<50%), moderate (50% to 70%), or severe (>70%) stenoses. There is no evidence of heart failure that would suggest therapy with ACE inhibitors, beta-blockers, and diuretics. The aortic valve leaflets appear thin and noncalcified, making aortic stenosis less likely. Cin CT images of the ventricles and aortic valve could be reconstructed, if desired, to assess ventricular function and leaflet mobility. Additional noninvasive testing is not indicated in this patient because of the abnormalities seen on coronary CTA.

Left anterior oblique cranial projection of the right coronary artery reveals a 60% to 70% stenosis in the midportion of the vessel (white arrow).

3. A 42-year-old man with diabetes and a family history of coronary artery disease undergoes coronary CTA after an equivocal exercise stress test. The following MPR image is obtained of the left main and left anterior descending (LAD) arteries. P.435

Curved MPR image of the LAD (white arrow).

Which of the following statements is true regarding the calcified plaque seen in the proximal LAD in this image? a. Additional postprocessing should be performed to remove the calcium blooming artifact. b. Coronary calcification may occur in the presence of atherosclerosis but is a nonspecific finding. c. Coronary calcification tends to overestimate coronary artery stenosis due to blooming artifact. d. The degree of coronary calcification correlates well with the severity of stenosis in the underlying vessel. View Answer Answer is c: Coronary calcification tends to overestimate coronary artery stenosis due to blooming artifact. This is due to attenuation (absorption) of the x-ray photons by deposits of calcium, which is relatively dense compared to its surrounding tissues. Currently, this artifact cannot be removed by postprocessing techniques. The presence of coronary calcification does correlate with an individual's overall atherosclerotic disease burden, but it does not predict the severity of stenosis of the underlying vessel. Calcification within an artery is a specific sign of atherosclerosis. 4. A 60-year-old man with a history of hypertension and dyslipidemia presents to the hospital in acute pulmonary edema approximately 72 hours after probable onset of an anterior myocardial infarction. The patient is stabilized and a cardiac catheterization is performed, which demonstrates a diffusely calcified 90% lesion of the ostial-proximal LAD that is not amenable to percutaneous intervention. The RCA and LCX arteries demonstrate mild to moderate diffuse disease. A transthoracic echocardiogram demonstrates a left ventricular ejection fraction of approximately 15%. He is referred for coronary artery bypass surgery and a cardiac MRI is obtained to assess for myocardial viability.

Delayed hyperenhanced image obtained 15 to 20 minutes after gadolinium DTPA, two-chamber view.

Delayed hyperenhanced image obtained 15 to 20 minutes after gadolinium DTPA, three-chamber view.

All of the following would be appropriate except: a. Surgical revascularization of the left anterior descending artery b. Medical therapy with an ACE inhibitor, beta-blocker, and diuretic

P.436

c. Medical therapy with aspirin and a statin d. Consideration for implantation of a defibrillator View Answer Answer is a: Surgical revascularization of the left anterior descending artery. Delayed hyperenhancement images demonstrate transmural scarring from the proximal to distal anterior and anteroseptal walls, as well as the apex and inferoapical segments. The transmural extent of hyperenhancement suggests a poor likelihood of recovery of myocardial function after revascularization (whether surgical or percutaneous), consistent with nonviable myocardium. Surgical revascularization would be high risk given his low ejection fraction, and unlikely to improve his long-term survival or ventricular function because of the nonviable myocardium in the infarct-related territory. The other choices would be indicated given the clinical scenario. 5. A 32-year-old woman is referred to your office for evaluation of occasional palpitations and increasing exertional dyspnea. She denies any history of fever, syncope, or neurologic deficits. Physical examination is unremarkable. A transthoracic echocardiogram demonstrates a poorly defined left ventricular mass, and a cardiac MRI is obtained for further evaluation of the lesion.

Still frame gradient echo image, three-chamber view.

Black blood (turbo spin echo) axial image depicting an intracardiac mass (white arrow).

Corresponding fat saturated black blood (T2-weighted STIR) axial image.

Both echo and MRI demonstrate normal systolic function and no valvular abnormalities. No other lesions are noted on the cardiac MRI study. A CT scan of the chest, abdomen, and pelvis are otherwise normal. Given the signal characteristics on the above image, this lesion most likely represents: a. Fibroma b. Lipoma c. Mxyoma d. Thrombus e. Papillary fibroelastoma View Answer Answer is b: Lipoma. In the first and second images, an encapsulated mass is visible in the posterolateral wall of the left ventricle. In the third image, the mass has similar signal intensity as the nearby subcutaneous fat, suggesting a possible fatty nature. This is confirmed on the subsequent fat-saturated black blood axial image, in which a special pulse is given prior to acquisition of the image to suppress signal arising from fatty tissue. The

mass now appears black due to loss of signal, as does the nearby subcutaneous fat, confirming the fatty nature of the mass. The fatty content of the mass and the normal left ventricular systolic function are not consistent with a left ventricular thrombus. Papillary fibroelastomas do occur on the endocardium but most often (50%) occur on the aortic valve. They are not usually encapsulated and often demonstrate a frondlike appearance (similar to pompoms used by cheerleaders) and frequently have a stalk. Myxomas are most often located in the atria and are not characterized by this degree of fat content. Many myxomas have patchy, dark areas of low signal intensity on MRI because of calcification within the tumor.

43 - Hallmarks of Essential and Secondary Hypertension


QUESTIONS 1. A 55-year-old white man is referred for evaluation of hypertension (BP 185/95 mm Hg), discovered during a blood pressure screening at his workplace. The patient states that he is well and has not seen a physician in many years. He describes himself as a fitness freak, as he is an active j ogger, abstains from alcohol, and limits his salt and fat intake. He denies any knowledge of hypertension, cardiovascular disease, renal disease, or diabetes mellitus. He takes no medications regularly. Family history is significant in that his father was known to be hypertensive and died of a stroke. His older brother is being treated for hypertension. On examination, the patient appears well, with a blood pressure of 178/96 mm Hg while seated and standing. Body weight is 71 kg (157 lb), and height is 178 cm (70 in). Optic fundus examination is significant for grade II hypertensive retinopathy. The remainder of the examination is normal. Complete blood count, electrolyte panel, blood urea nitrogen level, creatinine concentration, thyroidstimulating hormone level, and results of urinalysis are normal. Electrocardiography demonstrates normal sinus rhythm with left ventricular hypertrophy. To reduce the patient's cardiovascular morbidity and mortality, which therapy would you prescribe? a. Hydralazine b. Atenolol c. Losartan d. Doxazosin View Answer Answer is c: The educational objective of this question is to recognize the superiority of therapy with an angiotensin-receptor blocker over a traditional beta-blocker for cardiovascular morbidity and mortality in treating patients with primary hypertension and electrocardiographic evidence of left ventricular hypertrophy. Most previous antihypertensive trials that demonstrated reductions in cardiovascular morbidity and mortality were P.521

based on a stepped care approach using diuretics and beta-blockers. The recent Losartan Intervention for

Endpoint Reduction in Hypertension Study compared the angiotensin-receptor blocker losartan with the betablocker atenolol in patients with primary hypertension who had evidence of left ventricular hypertrophy. Despite similar reductions in blood pressure between the groups, losartan recipients had fewer primary cardiovascular events (death, myocardial infarction, or cerebrovascular accident), experienced a lower rate of new-onset diabetes mellitus, and tolerated the medication with fewer side effects (88). 2. A 51-year-old white man transfers to your practice after a change of insurance status. His medical history is positive for primary hypertension without target organ damage. He has no history of renal or prostatic disease. Laboratory values obtained from his former primary care physician show normal results for blood urea nitrogen, serum creatinine, electrolytes, urinalysis, prostate-specific antigen, and electrocardiography. He takes the -blocker doxazosin, 2 mg at bedtime. On examination, blood pressure is 152/93 mm Hg seated and standing. Body weight is 84 kg (185 lb). The remainder of the examination is normal. What is the appropriate course of action regarding the patient's antihypertensive therapy? a. Advise a low-sodium diet. b. Discontinue doxazosin therapy and consider an alternative agent. c. Advise high dietary intake of calcium and potassium. d. Increase the doxazosin to 4 mg a day. View Answer Answer is b: The objective of this question is to recognize that wit hdrawal of monotherapy with an -blocker is recommended to treat hypertension. The Antihypertensive and Lipid-Lowering Treatment to Prevent Heart Attack Trial documented an increased risk of cardiovascular events (especially congestive heart failure) with use of blockers. This adverse finding was published before completion of the full trial. The authors recommended that clinicians discontinue use of -blocker monotherapy for hypertension and consider alternative therapy. Use of blocker therapy in combination with other antihypertensive agents and as therapy for symptomatic benign prostatic hyperplasia were not precluded (89). 3. Which of the following statements about microalbuminuria is true? a. To be of clinical value, microalbuminuria must be measured in a timed 12- to 24-hour sample. b. Microalbuminuria is a cardiovascular risk factor that is independent of traditional Framingham risk factors. c. Microalbuminuria is present when the spot urine albumin-to-creatinine ratio is >500 mg/g. d. Microalbuminuria is a predictor of cardiovascular risk only in patients with diabetes. View Answer Answer is b: The educational objective is to identify microalbuminuria as a cardiovascular risk factor and appreciate its measurement in clinical practice. Further analysis of data from the Heart Outcomes Prevention Evaluation (HOPE) trial demonstrated that microalbuminuria was an independent predictor of cardiovascular events in both diabetic and nondiabetic persons at risk for such events.

Clinical measurement of microalbuminuria is an important tool for assessment of chronic kidney disease and estimation of cardiovascular risk. Recent guidelines from the National Kidney Foundation suggest that timed urine collections are not required and that a spot urine sample to calculate the albumin-to-creatinine ratio is preferred. The albumin-to-creatinine ratio varies by gender because of differences in muscle mass. The established criteria for albumin-to-creatinine ratios for normal, microalbuminuria, and overt clinical proteinuria are as follows: in men, a normal albumin-to-creatinine ratio is <17 mg/g, whereas in women, <25 mg/g is normal. In microalbuminuria, the ratio is 17 to 250 mg/g in men and 25 to 355 mg/g in women; in clinical proteinuria, the ratio is >250 mg/g in men, whereas in women, it is >255 mg/g (90,91). 4. A 37-year-old woman calls Monday morning seeking help with the worst headache ever Friday night and Saturday. The headache was associated with severe lethargy and intermittent confusion. She recovered and has felt well for the past 24 hours. She states that she does not have fever or neurologic or cardiovascular symptoms. Her medical history is significant for hypertension, and recurrent urinary tract infections related to her known autosomal dominant polycystic kidney disease. She is concerned because her father died of a stroke during dialysis. Her serum creatinine concentration is 2.6 mg/dL. Blood pressure at home currently is 146/92 mm Hg. What do you recommend for this patient? a. Arrange urgent magnetic resonance angiography of her head. b. Order computed tomography of her head without contrast. c. Arrange a consultation with the neurology/headache clinic. d. Make an office appointment for her to see you View Answer Answer is a: The educational objective is to recognize the presentation and diagnosis of berry aneurysm in a patient with autosomal dominant polycystic kidney disease. The patient's neurologic symptoms 48 hours earlier probably represent a sentinel bleed from a berry aneurysm. The likelihood of central nervous system bleeding after such an event is high and warrants urgent evaluation. Magnetic resonance angiography with gadolinium provides acceptable imaging of the carotids, circle of Willis, and central nervous P.522

system vasculature to identify or exclude berry aneurysm, which might require intervention. Overall, aneurysm is found in approximately 10% of all patients with autosomal dominant polycystic kidney disease and in about 24% of patients with polycystic kidney disease who have a positive family history of aneurysm. Routine screening with magnetic resonance angiography is often recommended for patients with this family history and autosomal dominant polycystic kidney disease. All patients with autosomal dominant polycystic kidney disease who have central nervous system symptoms should undergo magnetic resonance angiography evaluation to exclude a lifethreatening condition (92,93,94).

5. A 52-year-old rodeo rider is referred by his primary care physician for hypertension and hypokalemia over the past 6 months. Blood pressure and routine chemistries were normal last year at the time of a company physical. He has no history of cardiovascular disease, stroke, or renal disease. Family history is negative for hypertension. He uses alcohol socially and does not smoke, but chews tobacco. He takes no medications regularly. On examination the patient weights 77 kg (168 lb). Blood pressure is 184/102 mm Hg seated and standing. Except for trace pedal edema, the remainder of examination is normal. The primary care physician provides the following laboratory values: Blood urea nitrogen: 21 mg/dL Serum creatinine: 0.9 mg/dL Serum sodium: 141 mEq/L Serum potassium: 3.1 mEq/DL Serum chloride: 100 mEq/L Serum bicarbonate: 28 mEq/L A 24-hour urine test during salt loading reveals the following values: Creatinine: 1.1 g Sodium: 252 mEq Potassium: 128 mEq The daily aldosterone excretion rate is 6 mg (normal, 5 to 10 mg), plasma renin activity is 1 g/L/h, and plasma aldosterone level is 9 ng/dL. Which diagnostic test would you order next? a. Adrenocorticotropin hormone stimulation test b. Magnetic resonance angiography with gadolinium c. Serum cortisol and urinary free cortisol measurement d. Computed tomography of the adrenal glands View Answer Answer is c: The educational objective of this question is to recognize corticoid excess in a patient with hypertension and hypokalemia. This patient presents with hypertension, metabolic alkalosis with hypokalemia, and a low normal plasma and urinary aldosterone suggestive of corticoid excess due to tobacco chewing. Chewing tobacco is adulterated with licorice-containing glycyrrhizic acid. Licorice and its derivatives cause hypertension by inhibiting inactivation of cortisol by 11--dehydrogenase. This results in increased activation of corticosteroid receptors by cortisol, an effect that is most obvious for renal mineralocorticoid receptors, resulting in sodium retention and kaliuresis. Modest increases in the serum cortisol level and urinary level of free cortisol are diagnostic of the licoricecontaining products. The biochemical data do not support the diagnosis of primary hyperaldosteronism. The low plasma renin activity and clinical presentation do not suggest renal artery stenosis (95).

44 - Selected Topics in Pulmonary/Critical Care Medicine for an Intensive Review of Cardiology 45 - Twelve-Lead Electrocardiography 46 - Sudden Cardiac Death and Ventricular Tachycardia
1. Which of the following will most likely respond to verapamil? a. RVOT ventricular tachycardia b. Idiopathic left ventricular tachycardia c. Bundle branch re-entry tachycardia d. ARVD ventricular tachycardia View Answer Answer is b: Idiopathic left VT with RBBB morphology. This is a paroxysmal and sustained VT that occurs predominantly in en. It is a verapamil-sensitive fascicular VT and is the most common left VT. This type of VT is characterized by the following triad: (a) easily induced by atrial pacing, (b) RBBB morphology, and (c) absence of structural heart disease. 2. Which of the following ventricular tachycardias is most amenable to ablation? a. Ischemic cardiomyopathy VT b. ARVD VT c. Brugada syndrome VT d. Exercise-induced VT P.594

View Answer Answer is d: Exercise-induced ventricular tachycardia. This is an outflow-tract tachycardia. Mapping and ablation of the origin site is highly successful in abolishing this type of ventricular tachycardia. 3. Sudden cardiac death in patients >35 years old is most commonly associated with: a. Hypertrophic cardiomyopathy b. Coronary artery disease c. Long-QT syndrome d. Long QT secondary to various medications View Answer

Answer is b: Coronary artery disease. In patients >35 years old, the most common cause of VT is scar related to previous myocardial infarctions. 4. Bundle branch re-entry ventricular tachycardia is most commonly associated with: a. Enhanced automaticity in the right bundle b. Enhanced automaticity in the left bundle c. Supranormal conduction in the His bundle d. Abnormally slow conduction in the His bundle View Answer Answer is d: Abnormally slow conduction in the His bundle sets up the conditions of re-entry required to sustain this kind of tachycardia. 5. The following ECG is consistent with:

a. Acute anteroseptal myocardial infarction b. Abnormal SCN5A channel c. Abnormal KCQN1 channel d. Old anteroseptal myocardial infarction with an aneurysm View Answer Answer is b: Abnormal SCN5A channel, which causes the Brugada syndrome.

47 - Electrophysiologic Testing Including His Bundle and Other Intracardiac Electrograms


1. Where is the site of the block?

a. AV node b. Infra-His c. Intra-His d. AV node and infra-His View Answer Answer is b: The tracing shows atrial pacing with RBBB and second-degree AV block without prolongation of the PR or AH intervals prior to the blocked beat (third paced beat). On this third paced beat, the His electrode shows an atrial electrogram followed by a His deflection, but no subsequent ventricular electrogram or QRS. Thus, the block occurs below the bundle of His (infra-Hisian block). P.622

2. Where is the site of the block?

a. AV node b. Infra-His c. Intra-His d. AV node and infra-His View Answer Answer is a: The tracing shows atrial pacing (S1 drive) with 2:1 AV block. Inspection of the His bundle electrogram tracings demonstrate S1 atrial pacing stimuli flowed by atrial electrograms. After the first paced beat, there is a His bundle electrogram followed by a ventricular electrogram and QRS on the surface ECG. After the second paced beat, no His bundle electrogram follows the atrial electrogram. The next paced beats repeat this pattern. The block is at the level of the AV node, because conduction is blocked prior to arrival to the His bundle. 3. Where is the site of the block?

a. AV node b. Infra-His c. Intra-His d. AV node and infra-His View Answer Answer is d: This tracing shows second-degree AV block during atrial pacing. The His bundle electrogram demonstrates the atrial pacing stimuli followed by atrial electrograms. After the first atrial paced beat, there is a long AH interval followed by a His electrogram, but no ventricular electrogram or QRS. This beat blocks below the bundle of His. After the second paced beat there is a slightly longer AH interval followed by a ventricular electrogram on the RVA tracing and a corresponding surface QRS. After the third paced beat, the AH is longer still but there is no conduction after the His electrogram to the ventricles. This beat again shows infra-Hisian block. After the fourth paced beat, there is no His electrogram. This beat blocks in the AV node and the series shows AV node Wenckebach occurring (gradually prolonging AH interval followed by block in the AV node). The fifth paced

beat shows conduction after the block with a shorter AH interval followed by conduction to the ventricles. The sixth paced beat shows a small His deflection with slightly longer AH, but infra-Hisian block (no ventricular activation). The seventh paced beat shows a slightly longer AH interval with conduction to the ventricles. Thus, the tracing demonstrates two levels of block, in the AVN (Mobitz I Wenckebach pattern) and infra-Hisian block. 4. What is the diagnosis?

a. Orthodromic AVRT b. Left-sided accessory pathway c. Atrial tachycardia d. AV node re-entrant tachycardia View Answer

Answer is d: The tracing shows a narrow QRS-complex tachycardia with a cycle length of 350 milliseconds. The coronary sinus atrial electrograms show a concentric atrial activation pattern (earliest at CS 7-8 at the septum and later at more distal CS electrodes) with near simultaneous activation of the atrium and ventricle. The earliest atrial activation is likely the small deflection at the onset of the QRS on the HBE tracing, which actually slightly precedes the ventricular activation. This pattern is consistent with AV node re-entrant tachycardia. 5. What is the diagnosis?

a. Left-sided accessory pathway b. Right-sided accessory pathway c. AV node reentrant tachycardia d. Sinus tachycardia P.623

View Answer Answer is a: This tracing shows a narrow complex tachycardia with a cycle length of 370 milliseconds. The anterograde activation occurs via the AV node and His-Purkinje system (AH seen in HBE1-3 with narrow QRS). The earliest atrial activation occurs in the distal CS as CS 1-2. This eccentric activation pattern indicates retrograde activation via a left lateral accessory pathway. The tachycardia is consistent with orthodromic atrioventricular reentrant tachycardia using a retrogradely conducting left-sided accessory pathway.

48 - Supraventricular Tachycardias
1. A 44-year-old patient with no previous cardiovascular history, who presents with a wide-QRS, irregular, and fast tachycardia (on ECG) is best treated with: a. Lidocaine b. Procainamide c. Metoprolol d. Diltiazem View Answer Answer is b: This patient has atrial fibrillation with pre-excited QRS and should be treated with procainamide. AV node-blocking agents are absolutely contradicted because they will favor conduction over the accessory pathway with increased risk of degeneration into ventricular fibrillation. 2. Catheter ablation is an established and wellaccepted treatment for each of the following tachycardia except: a. AV node re-entry b. AV re-entry c. Permanent junctional tachycardia d. Sinus tachycardia View Answer Answer is d: Catheter ablation is an established therapy for all the arrhythmias listed except sinus tachycardia. 3. Which of the following forms of congenital heart disease is commonly associated with Wolff-ParkinsonWhite syndrome? a. Aortic stenosis b. Ebstein anomaly c. Pulmonary stenosis d. Atrial septal defect

View Answer Answer is b: Ebstein Anomaly is associated with WPW in 6% of patients with this congenital anomally. 4. Which of the following supraventricular tachycardias is associated with tachycardia-induced cardiomyopathy? a. Permanent junctional tachycardia b. Incessant atrial tachycardia c. Atrial flutter with rapid ventricular response d. All of the above View Answer Answer is d: All tachycardias of incessant nature can cause tachycardia-induced cardiomyopathy. 5. Which test would you consider for an asymptomatic 31-year-old man with intermittent pre-excitation? a. Holter monitoring b. Electrophysiologic study c. Exercise test d. Catheter ablation e. None of the above View Answer Answer is e: No further investigation or treatment is indicated for an asymptomatic patient with intermittent preexcitation. 6. Conduction block in the AV node without termination of the tachycardia is compatible with all of the following mechanisms except: a. Atrial tachycardia b. AV re-entry tachycardia c. Atrial flutter d. Sinus tachycardia View Answer Answer is b: The only tachycardia that cannot sustain with conduction block in the AV node is AV re-entry. P.638

7. The initial manifestations of Wolff-Parkinson-White syndrome include which of the following? a. Atrial fibrillation b. AV re-entry tachycardia c. Ventricular fibrillation d. Wide-QRS tachycardia e. All of the above

View Answer Answer is e: All of the above are possible arrhythmias in Wolff-Parkinson-White syndrome. 8. Administration of metoprolol is more likely to terminate: a. Sinus tachycardia b. Atrial tachycardia c. Atrial fibrillation d. AV re-entry tachycardia View Answer Answer is d: Sinus tachycardia will slow down but not terminate. Atrial tachycardia and atrial fibrillation will not be affected by metoprolol. 9. Which of the following is the treatment of choice to terminate a narrow-QRS tachycardia? a. Metoprolol b. Diltiazem c. Adenosine d. Procainamide e. Cardioversion View Answer Answer is c: Adenosine is the best acute treatment for narrow-QRS tachycardia. 10. For a patient with Wolff-Parkinson-White syndrome who presents with a regular wide-QRS tachycardia, all of the following are possible treatment choices except: a. Procainamide b. Cardioversion c. Amiodarone d. Ibutilide e. Adenosine View Answer Answer is e: Adenosine and AV node-blocking agents are contraindicated in pre-excited arrhythmias. 11. Transesophageal recording may help in establishing the diagnosis in which of the following supraventricular tachycardias? a. Atrial tachycardia b. AV node re-entrant tachycardia c. AV re-entrant tachycardia d. Atrial flutter e. All of the above View Answer

Answer is e: Transesophageal recording can provide information that is helpful in establishing a diagnosis in all of the supraventricular tachycardias listed. 12. Change in the tachycardia rate with development of bundle branch block is consistent with: a. Atrial tachycardia b. AV node re-entry c. AV re-entry d. Sinus tachycardia View Answer Answer is c: AV re-entry due to an accessory bypass tract ipsilateral to the bundle branch block is the only arrhythmia associated with the above behavior. 13. The presence of an r prime in V1 during narrow-QRS tachycardia is suggestive of: a. AV re-entry b. AV node re-entry c. Rate-dependent bundle branch block d. Atrial tachycardia View Answer Answer is a: The correct answer is AV node re-entry. 14. The best therapy for multifocal atrial tachycardia is a. Digoxin b. Diltiazem c. Metoprolol d. Flecainide e. Treatment of the underlying disorder View Answer Answer is e: No drug therapy will be effective for multifocal atrial tachycardia if the underlying disorder is not corrected.

49 - Atrial Fibrillation and Flutter 50 - Wide-Complex Tachycardia: Ventricular Tachycardia versus Supraventricular Tachycardia
Wide-Complex Tachycardia: VT versus SVT Worksheet 1. QRS-Complex Duration VT: QRS > 140 ms for RBBB, QRS > 160 ms for LBBB 2. QRS-Complex Axis

VT: right superior 3. Capture and Fusion Complexes 4. QRS Precordial Concordance 5. WCT: Brugada's Criteria I

Step 1. Absence of an RS complex in all precordial leads? Step 2. R-to-S interval > 100 ms in one precordial lead? Step 3. AV dissociation? Step 4. Morphology criteria for VT present in both V1-V2 and V6?

RBBB morphology

VT

SVT

V1

Monophasic R

V1

Triphasic

R (>30 ms) + any S qR

rSR', rSr' rR', rsr'

V6

RS (R < S) QS, Qrs

V6

Triphasic Rs, RS (R > S)

LBBB morphology

VT

SVT

V1

rS: Broad r > 30 ms Notching/delay in S QS 70 ms RT taller than RS

V1

rS, QS (rapid downstroke)

V6

QR, QS, QrS, qR Rr'

V6

rR' Monophasic R No Q waves

6. WCT: Brugada's Criteria II

Step 1. Predominantly negative QRS complex in precordial leads V4-V6? Step2. Presence of a QR complex in one or more of leads V2-V6? Step3. Atrioventricular dissociation?

7. Diagnosis: ECG Criteria Miscellaneous Conditions


8. Special Cases

QRS complex during WCT narrower than during NSR: suggests VT. Contralateral BBB in NSR and WCT: suggests VT. Rapid irregular WCT + beat-to-beat QRS-to-QRS interval variation: atrial fibrillation with WPW.

Misclassification of SVT for VT: Pre-excited tachycardia, paced ventricular rhythm. Misclassification of VT for SVT: BBR-VT (without evidence of AV dissociation). Narrow QRS VT: ILV-VT

51 - Pacemakers and Defibrillators


1. The AV delay is 200 milliseconds and the time from a paced QRS to the next atrial paced event is 800 milliseconds. To what basic or lower rate has the pacemaker been programmed? a. 100 beats/min b. 90 beats/min c. 80 beats/min d. 70 beats/min e. 60 beats/min View Answer Answer is e: The cycle length consists of the AV interval + VA interval. These two intervals added together and converted to a heart rate yield the lower rate or base rate programmed for this pacemaker patient. 200 ms + 800 ms = 1,000 ms = cycle length Base heart rate = 60,000/1,000 ms = 60 beats/min P.684

Note that the AV interval can be dynamically shortened (based on the sensor and or the atrial rate) and there can be a shortening of the AV interval if there is a sensed P wave instead of an atrial paced beat. In addition, the rate of the pacemaker can be increased with apparent increases in the base rate if the sensor detects a need to increase the paced rate. 2. A 20-year-old college student loses consciousness during math class. She has had an average of two syncopal spells a year for as long as she can remember. She has no warning before her spells, and she has dislocated her thumb and chipped a tooth in the past during similar spells. The patient's indication for a pacemaker is a. Class I b. Class II c. Class III View Answer Answer is b: Despite the controversy about the use of pacemakers for vasovagal (neurocardiogenic) syncope, it is appropriate to implant a pacemaker in this situation. The Vasovagal Syncope Pacemaker (VPS) study demonstrated a marked reduction in syncope in patients with at least six syncopal spells in their lifetime. Clearly, this patient has a history consistent with neurocardiogenic syncope with severe spells. The VPS study required a tilt study with relative bradycardia. There did not need to be asystole. 3. The sensed AV interval is 150 milliseconds and the PVARP (postventricular atrial refractory period) is 350 milliseconds. What is the most rapid atrial rate that the pacemaker can track 1:1? a. 300 beats/min

b. 250 beats/min c. 200 beats/min d. 150 beats/min e. 120 beats/min View Answer Answer is e: The maximal rate at which a DDD pacemaker can track an atrial rhythm is limited by the shortest interval in which the atrium can be detected. By adding together the PV interval (the AV interval initiated by a P wave and terminated with a ventricular pacemaker output) and the PVARP (the time during which the atrium cannot sense another P wave after a ventricular sensed or paced event), the total refractory period can be calculated. That interval converted to a heart rate is the maximal rate at which the pacemaker can participate in producing a paced rhythm. Both the PV delay and the PVARP can vary on the basis of atrial rate and sensor rate, but in this example the intervals are fixed. Thus, AV interval + PVARP = 150 ms + 350 ms = 500 ms Converting this to heart rate, we see that the maximal paced rate = 60,000/500 milliseconds = 120 beats/min. To increase the upper tracking rate it would be necessary to shorten the sensed AV interval, the PVARP, or enable rate-adaptive shortening of these intervals. 4. A pacemaker-dependent 65-year-old woman says that her activity-sensing pacemaker programmed to the DDDR mode (unipolar) makes her heart race every time she sweeps the floor. Which of the following could remedy the patient's situation? a. Increase the rate response slope. b. Program the atrial channel to bipolar paced configuration. c. Program the ventricular channel to bipolar sensed configuration. d. Decrease the atrial sensitivity from 1 mV to 3 mV. e. Decrease the ventricular sensitivity from 1 mV to 4 mV. View Answer Answer is d: This woman is using her upper body, which has the potential to activate her activity sensor and to produce myopotentials from use of the pectoralis major muscle. Increasing the rate response slope will increase the heart rate increase related to her activity. Programming the atrium to bipolar paced configuration would not affect her heart rate but could help if her complaint was secondary to stimulation of the pectoral muscles. Programming the ventricle to bipolar sensed configuration would reduce the likelihood that the ventricular channel would detect myopotentials, because the muscle would no longer be within the antennae being sensed by the ventricle. Sensed events on the ventricular channel would inhibit ventricular output and could be responsible for syncope due to bradycardia. Decreasing the ventricular sensitivity would potentially decrease the likelihood that myopotentials would be sensed, but this would cause inhibition of the ventricular output and a decreased heart rate. Decreasing the atrial sensitivity from 1 mV to 3 mV will likely decrease the probability of sensing

myopotentials on the atrial channel. These atrial sensed events would have been tracked to the ventricle and cause the patient to perceive a tachycardia rhythm. 5. The VVI pacemaker is programmed to a lower rate of 80 beats/min. There are PVCs and usually the heart rate is paced at 80 beats/min, but intermittently there are intervals between intrinsic R waves of 960 milliseconds. Which of the following could be the explanation for the electrocardiographic findings? a. Paced bipolar impedance of 300 ohms b. Hysteresis rate of 60 beats/min c. Sleep rate of 55 beats/min d. Paced unipolar impedance of 250 ohms e. PVC response View Answer Answer is b: The rate of 80 beats/min needs to be converted to an interval. Cycle length = 60,000/80 = 750 milliseconds. The interval of 960 is longer than the 750 (80 beats/min). Longer intervals than the lower rate (base rate) of the pacemaker suggest (a) inhibition, (b) failure of output by the pacemaker (lead or generator related), or (c) an algorithm that explains the particular circumstance. A paced bipolar impedance of 300 ohms is relatively low, but normal. If this represented a marked drop from previous measurements, then it is possible that there is a short within the pacing lead. The hysteresis rate of 60 beats/min translates to a sensed P.685

escape interval of 1,000 milliseconds. This could explain the ECG as long as the paced intervals between ventricular events were 750 milliseconds. If the ECG findings occurred only at night, then intervals of 960 milliseconds would be normal, but would not explain the other paced intervals at 750 milliseconds. A unipolar pacing impedance cannot be too low to work. This is low but does not explain the findings. PVC responses can extend the PVARP to avoid initiating a pacemaker-mediated tachycardia, but would not change the escape interval of the pacemaker.

52 Syncope
1. An 83-year-old man comes to see you in your office complaining of three episodes of abrupt loss of consciousness in the last year. His PMH is negative. His internist recently performed a stress echo test and Holter monitoring that were normal. His ECG reveals trifascicular block. What is the next step? a. Perform EPS, and if negative, implant a Reveal device. b. Perform EPS, and if negative, implant an ICD. c. Consider EPS for VT, and if negative, recommend a pacemaker. d. Schedule a tilt test.

View Answer Answer is c: The presence of trifasicular or bifasicular block on the ECG suggests that the underlying etiology of syncope may be intermittent heart block, Mobitz type II, third-degree heart block, so-called Stokes-Adams block. Given a normal stress echo test and normal LV function, electrophysiologic testing will likely be negative for ventricular tachycardia. Based on current ACC/AHA guidelines, when no other cause for syncope is found, implantation is indicated for syncope that has not been demonstrated to be due to AV block. 2. A 57-year-old man with dilated cardiomyopathy presents to the emergency department with a facial laceration. He reports that he was urinating during the night and suddenly lost consciousness, falling and sustaining the injury. He felt fine before and after the event. He has an EF of 25% secondary to probable viral myocarditis. His ECG reveals IVCD and occasional multifocal PVCs. What is the next step? a. Discharge from the emergency department with a 48-hour Holter monitor. b. Admit, perform EPS, and if negative, implant a Reveal device. c. Schedule an outpatient tilt test. d. Admit, perform EPS, and if negative, offer an ICD. View Answer Answer is d: Syncope in a patient with dilated cardiomyopathy is a very poor prognostic sign. Electrophysiologic testing has a low negative predictive value and therefore cannot be wholly relied on to screen patients who need a defibrillator. Implantation of a defibrillator remains a Class 2B indication in the presence of advanced P.703

structural heart disease denoting severe ischemic or nonischemic cardiomyopathy. In addition, based on the ejection fraction alone, the patient qualifiesy for ICD implantation according to the recent Definite and SCD-Heft data. 3. A 21-year-old female college student presents to her local emergency department because she fainted twice earlier that day. She reports that the first episode occurred while she was in the shower. It was preceded by nausea with diaphoresis, followed by sudden loss of consciousness. After she awoke on the floor, she felt very nauseated, diaphoretic, and vomited. She tried to stand but fainted again. She is otherwise healthy but has had the flu for 3 days. You are asked to consult. Her PMH, ECG, physical exam, and lab are normal. She had previously fainted once, while donating blood at a blood drive. What tests should you order? a. Tilt test b. Holter monitoring c. Stress echo test d. None of the above View Answer Answer is d: Patients who present with a typical vasovagal episode with a classic prodrome and sequalae, who are otherwise healthy, probably require no other diagnostic testing or therapy, as the most unlikely etiology is

vasovagal syncope. Tilt table testing is indicated only if the syncope becomes recurrent, or after single episodes of syncope with atypical features or for a high-risk patient. 4. Treatment for vasovagal syncope usually involves avoiding offending stimuli, dehydration, and prolonged standing; improving or decreasing venous pooling; and high-salt and high-fluid diet. For recurrent episodes, pharmacologic therapy is often employed. Initial pharmacologic therapy consists of all of the following except: a. Beta-blockers b. Disopyramide c. Serotonin reuptake inhibitors d. Florinef View Answer Answer is b: Disopyramide (Norpace) is a Type 1A sodium channel antiarrhythmic medication. It was proposed to be effective for vasovagal syncope based on its negative inotropic and anticholinergic effects. However, in a very well designed study, using disopyramide loading and repeat tilt table tests, no efficacy was found. In addition, there is genuine concern for proarrhythmia in using the antiarrhythmic agents for treatment of a relatively benign disorder. Therefore, disopyramide may have a role for some patients, but it should not be used as initial therapy. 5. Carotid sinus syndrome is probably the second most common cause of neurally mediated syncope. It results from hypersensitivity of the carotid reflex and causes marked bradycardia and, frequently, concominent hypotension. All of the following are true regarding the features and treatment of carotid hypersensitivity except: a. The finding of carotid sinus hypersensitivity is extremely specific for the presence of carotid sinus hypersensitivity and syndrome, and mandates pacemaker implantation. b. Pacemaker implantation has been shown to significantly reduce the number of syncopal spells. c. Patients with recurrent unexplained falls or injuries should be considered to have a neurally mediated syncopal etiology such as carotid sinus hypersensitivity and be tested either with carotid sinus massage testing or tilt table testing. d. Carotid sinus hypersensitivity syndrome may result from abnormal proprioception and barrel receptor responses, in the carotid artery and the surrounding sternocleido mastoid. View Answer Answer is a: Although the finding of carotid sinus hypersensitivity in a patient with recurrent syncope is highly suggestive, without the presence of the clinical syndrome the finding is relatively nonspecific. Carotid sinus hypersensitivity has been shown to be prevalent in patients with coronary disease and other forms of atherosclerotic disease as well. The sine qua non for carotid sinus syndrome is demonstration of carotid sinus hypersensitivity during carotid sinus massage, and a clinical scenario consistent with syncope resulting from direct stimulation of the carotid sinus baro and vagal reflex.

53 - Pericardial Diseases

1. What is the most common cause of constrictive pericarditis in the United States? a. Previous cardiac surgery b. Irradiation c. Tuberculosis d. Idiopathic or viral View Answer Answer is d: Idiopathic or viral. In one series (Bertog et al), etiology was idiopathic in 46%, previous cardiac surgery in P.712

37%, irradiation in 9%, and miscellaneous (including tuberculosis) in 8% of the patients. 2. A young patient presents with chest pain and ankle swelling. Four weeks earlier, the patient had been diagnosed with viral pericarditis and treated with Ibuprofen. Presently he has no chest pain or shortness of breath. An echocardiogram reveals new findings consistent with mild constriction. What is the next course of management? a. MRI to better assess pericardium b. Consultation for pericardiectomy c. Start steroids d. NSAIDS and Colchicine View Answer Answer is d: NSAIDS and Colchicine. Acute pericarditis can be followed by mild transient constriction, which usually resolves in 2-3 months. Steroids could be used for resistant cases. Pericardiectomy is not indicated. 3. Pulsus paradoxus can be caused by all of the following except: a. Pericardial tamponade b. Asthma c. Obesity d. Constrictive pericardial disease View Answer Answer is d: Constrictive pericardial disease. Pulsus paradoxus is an inspiratory fall in systolic blood pressure > 10 mmHg. It occurs most often in pericardial tamponade, asthma, and obesity. It is not usually seen in constriction unless there is an effusive component. 4. Central venous pressure examination in tamponade reveals: a. Prominent X descent (rapid ventricular filling during systole) and absent Y descent (absent diastolic filling). b. Prominent X and Y descents. c. Prominent Y but blunted X descent. d. These waveforms can only be discerned with right heart catheterization.

View Answer Answer is a: Prominent X descent (rapid ventricular filling during systole) and absent Y descent (absent diastolic filling). 5. Post-MI pericarditis is associated with all the following except: a. Incidence has been reduced by thrombolytic therapy. b. Most common after inferior MI. c. ECG manifestations are rare. d. Associated with atrial arrhythmia. View Answer Answer is b: Post-MI pericarditis occurs more commonly after larger infarcts, usually anterior MI.

54 - Effects of Systemic Diseases on the Heart and Cardiovascular System


1. Match the syndrome with the phenotype:
a. Marfan 1. Basal cell nevi, deafness, pulmonic stenosis

b.

Osler-Weber-Rendu

2.

Infundibular stenosis, atrial septal defect, patent ductus arteriosus, tetralogy of Fallot

c.

Kartagener

3.

Mitral valve prolapse, aortic regurgitation, aortic aneurysm/dissection

d.

Noonan

4.

Paradoxical emboli

e.

LEOPARD

5.

Elfin facies, hypercalcemia, supravalvular aortic stenosis, atrial septal defect, ventricular septal defect

f.

Williams

6.

Dextrocardia, chronic sinusitis, infertility

View Answer Answers: a-3, b-4, c-6, d-2, e-1, f-5 2. In a patient with myotonic dystrophy, which of the following is not an associated electrocardiographic abnormality? a. Intraventricular conduction delay b. Left ventricular hypertrophy by voltage criteria c. Pathologic Q waves d. Atrioventricular conduction block

View Answer Answer is b: Unlike the muscular dystrophies, myotonic dystrophy is not associated with a hypertrophic cardiomyopathy. It is, however, associated with the electrocardiogram abnormalities above. 3. Typically, Guillain-Barr syndrome is not associated with which of the following abnormalities? a. Hypertension b. Sinus tachycardia c. Sinus bradycardia d. Orthostatic hypotension e. Ventricular tachycardia View Answer Answer is c: The autonomic instability that characterizes the polyneuropathy in Guillain-Barre syndrome may result in hypertension, orthostatic hypotension, and sinus tachycardia; P.721

sinus bradycardia is usually not observed. Ventricular tachycardia may also occur. 4. A 35-year-old woman with rheumatoid arthritis (RA) presents for evaluation of hypertension and resting tachycardia. Her RA is controlled on 5 mg/day of prednisone. She does not experience chest pain or dyspnea. Her vital signs include an irregular apical pulse of 120 beats/min and a blood pressure of 165/70 mm Hg. Serum electrolytes and a complete blood count are within normal limits. What is the most appropriate initial therapy? a. Increase the steroid dosage. b. Discontinue steroid therapy. c. Transsphenoidal pituitary resection followed by corticosteroid and T4 replacement. d. -Adrenergic blockers and methimazole. View Answer Answer is d: This patient has hyperthyroidism. Her rheumatoid arthritis increases her risk of other autoimmune disorders; the most common cause of hyperthyroidism is Graves disease, an autoimmune disorder characterized by autoantibodies against the thyrotropin receptor. Her associated hypertension and atrial fibrillation are best managed initially with -adrenergic-blocking agents, then by antithyroid medications. 5. A 27-year-old white man presents to the outpatient department with dyspnea on exertion. He has no symptoms of chest pain or presyncope, and reports only a migratory joint discomfort on review of systems. Vital signs demonstrate a heart rate of 90 beats/min and a blood pressure of 125/44 mm Hg. His jugular venous pressure is normal, his lungs are clear, and his heart rhythm is regular. What findings would you expect on further examination and testing? a. Malar rash, holosystolic murmur radiating to the axilla, positive serum anti-dsDNA antibody b. Telangiectasias, sclerodactyly, diffuse ST-segment elevations on electrocardiogram

c. Loss of lumbar lordosis, diastolic murmur at the left sternal border, iritis, positive serum HLA-B27 marker d. Digital ulnar deviation, positive serum rheumatoid factor, electrical alternans on electrocardiogram e. Atrioventricular conduction block and a restrictive pulmonary function test pattern View Answer Answer is c: This patient has ankylosing spondylitis. The wide pulse pressure results from a regurgitant aortic valve, which would also cause the diastolic murmur. The arthritis associated with ankylosing spondylitis is asymmetric and migratory, and loss of lumbar lordosis occurs with sacroiliitis. The clinical findings listed would not account for scleroderma-associated pericarditis (as in choice b), for rheumatoid arthritis-associated pericardial effusion (as in choice d), or for sarcoid-associated heart block (as in choice e). 6. A 45-year-old woman with a history of Hodgkin disease presents with dyspnea. She underwent chemotherapy and external-beam radiation 10 years ago, and her Hodgkin disease has been in clinical remission ever since. Which of the following is not likely to be the cause of her dyspnea? a. Complete heart block b. Coronary artery disease c. Aortic stenosis d. Constrictive pericarditis View Answer Answer is a: Complete heart block is not a complication of external-beam radiation. Diffuse coronary artery disease, valvulitis causing aortic stenosis, and constrictive pericarditis may all result from radiation-induced mediastinal damage. 7. Which of the following is not a cardiac complication of HIV infection and antiretroviral therapy? a. Diffuse coronary artery disease b. Intraventricular conduction delay c. Pericardial effusion d. Dilated cardiomyopathy with regional wall motion abnormalities View Answer Answer is b: Although HIV and protease inhibitors may cause diffuse coronary artery disease, pericardial effusion, and focal myocarditis, intraventricular conduction delay is not an associated complication of the infection or therapy.

55 - Pregnancy and Heart Disease


(Adapted from the syllabus of The Cleveland Clinic Intensive Review of Medicine, Drs. Raymond and Maroo, year 2005.)

1. All of the following lesions are contraindications to pregnancy except: a. Primary and secondary pulmonary hypertension b. Shunt lesions complicated by Eisenmenger syndrome c. Mild to moderate mitral stenosis d. Complex cyanotic congenital heart disease e. Diminished residual left ventricular function (ejection fraction <50%) View Answer Answer is c: Listed in this question are four clear contraindications to pregnancy, including primary and secondary pulmonary hypertension; shunt lesions complicated by Eisenmenger syndrome, because of the high pulmonary pressures and right-to-left shunting; complex cyanotic congenital heart disease; and residual poor left ventricular function following peripartum cardiomyopathy and other cardiomyopathies. Mild to moderate mitral stenosis is usually well tolerated during pregnancy and can be treated with percutaneous mitral valvuloplasty in the event the patient becomes dyspneic in the second or third trimester. P.733

2. A 28-year-old G3P2 Hispanic woman with known mild mitral stenosis presents to your office after referral by her obstetrician. She is now 18 weeks pregnant and has noted 2 weeks of progressive dyspnea on exertion, which she states is much worse than during her last pregnancy. During physical examination you note 3 cm jugular venous distension at 45 degrees, rales halfway up the posterior lung fields, and a Grade II-III diastolic rumble murmur at the apex. An echocardiogram is consistent with moderate to severe mitral stenosis. All of the following therapeutic modalities are appropriate except: a. Immediate admission to the hospital with plans for urgent open mitral commissurotomy. b. Initial admission to the hospital including bedrest in the left lateral decubitus position and gentle diuresis. c. Consider percutaneous mitral valvuloplasty if initial conservative measures such as bedrest and diuresis do not result in complete resolution of her symptoms. d. The initiation of oral digoxin as prophylaxis against atrial fibrillation. View Answer Answer is a: Pregnancy complicated by symptomatic mitral stenosis is first managed by bedrest and general diuresis as well as the initiation of digoxin for atrial fibrillation prophylaxis. Percutaneous mitral valvuloplasty should be considered if symptoms do not resolve with initial measures. Open mitral commissurotomy is seldom necessary in the modern day of valvuloplasty. 3. A 37-year-old woman presents with dyspnea. She is 2 weeks postpartum status post-uncomplicated delivery of her first child. Physical examination findings include a heart rate of 100 beats/min and prominent jugular venous distention. Inspiratory rales are noted at the lung bases. The apical impulse is diffuse and displaced. Also noted is a third heart sound. On echocardiography, the left ventricle is moderately enlarged and systolic

function is reduced, with an estimated ejection fraction of 25%. Mild mitral regurgitation is noted. All the following statements about this patient's condition are true except: a. Predisposing factors include the patient's age and that this was her first pregnancy. b. Future pregnancies are contraindicated if the left ventricular ejection fraction does not return to 50% in 6 months. c. Management should be conservative and medical therapy should be instituted immediately. d. Recurrent problems occur in approximately 50% of subsequent pregnancies. e. There is a 50% chance of complete resolution of symptoms and left ventricular dysfunction with therapy. View Answer Answer is a: This patient with obvious peripartum cardiomyopathy should be treated conservatively with medical treatment. Risk factors for peripartum cardiomyopathy include a multiparous female, advanced maternal age, twin pregnancy, previous history of peripartum cardiomyopathy, and slight increase with African American ethnicity. Approximately 50% of patients experience complete resolution of left ventricular dysfunction. If left ventricular ejection fraction does not return to >50%, the patient is at higher risk for subsequent congestive heart failure during another pregnancy and should be so advised. 4. A 29-year-old G2P1 with a mechanical mitral valve prosthesis presents with rapidly progressive dyspnea in her 28th week of gestation. She takes coumadin with an international normalizing ratio (INR) of 1.8. Her physical exam reveals a heart rate of 100 beats/min, prominent jugular venous distention, rales up the back, and a displaced apical impulse. The mechanical prosthetic sounds are blunted, in addition to a soft systolic murmur. Echocardiography revealed an 18-mm transvalvular mitral gradient (5 mm prior to pregnancy) and mechanical leaflets that were difficult to visualize. Left ventricular function was normal. All of the following are appropriate treatment options except: a. Cardiac catheterization to measure the mitral valve gradient and coronary angiography b. Emergency high-risk obstetric evaluation c. Thrombolytic therapy d. Emergency cardiovascular surgery e. Administration of both furosemide and heparin View Answer Answer is a: This emergency situation needs to be evaluated by a cardiac surgeon and high-risk obstetrics specialist. Thrombolysis should be considered emergently unless the patient is at a facility where cardiac surgery is an option. A transesophageal echocardiogram is preferable if it can be performed safely. However, the entire history and physical scenario is consistent with thrombosis of a mechanical prosthesis. Urgent intervention is necessary to save both the mother and fetus. Mild diuresis may be helpful initially. Thrombolysis has been shown to be safe during pregnancy, without adverse effects on the mother or fetus in this emergency situation. If necessary, emergency cardiovascular surgery can be performed, but with an adverse risk primarily to the fetus. Perioperative uterine and fetal heart tone monitoring is necessary.

5. A 32-year-old white woman with suspected Marfan syndrome presents for consultation P.734

during the first trimester of her first pregnancy. All of the following statements are true except: a. There is a nearly 50% risk that her child will inherit this syndrome. b. Compared to the nonpregnant state, she has an increased risk of aortic rupture and dissection during this gestation. c. Her risk of aortic dissection increases as the ascending aorta exceeds 4 cm in diameter. d. Natural childbirth via vaginal delivery with a local anesthetic is appropriate with an aortic root diameter of 5.5 cm. View Answer Answer is d: All of the statements are true except d. Natural childbirth via vaginal delivery is appropriate if the aortic root is <4 cm; otherwise, this patient should be delivered by cesarean section as soon as the fetus is judged to be mature. The risk of aortic rupture or dissection increases substantially >4 cm diameter in the presence of the stress of vaginal delivery.

56 - Women and Heart Disease


1. All of the following are true regarding women with a myocardial infarction except: a. Women are more likely to have complications such as VSD, heart failure, cardiac rupture, and shock than men. b. Women are more likely than men to have had a prior myocardial infarction. c. Women have unadjusted 30-day and 1-year mortality rates post-MI that are higher than those for men. d. Primary PTCA decreased the risk of intracranial hemorrhage that is seen with thrombolytics. View Answer Answer is b: Women who present with an MI are less likely to have had a prior MI, and are more likely to have mechanical complications including VSD, congestive heart failure, cardiac rupture, and shock. Women also have higher 30-day and 1-year unadjusted mortality rates compared to men who present with an MI. Primary PTCA has decreased the risk of intracranial hemorrhage that is seen with thrombolytics. 2. Women with hypertension are a. Less likely than men to be aware of their diagnosis b. More likely than men to reach their target blood pressure c. Less likely than men to have a CVA than men with hypertension d. Less likely than men to have renovascular hypertension View Answer

Answer is b: Women with hypertension are more likely than men to be aware of their diagnosis, to have appropriate treatment, and to reach their target blood pressure. Women with hypertension are more likely to have a stroke than men with hypertension. Women have renovascular hypertension much more commonly than men (ratio 8:1). 3. All of the following are true regarding diabetes as a cardiac risk factor in women except: a. Eliminates the 10-year gender gap b. Increases the risk of CAD fivefold over women without diabetes c. Increases the risk for developing congestive heart failure post-MI d. Increases the risk of MI 10-fold over women without diabetes View Answer Answer is d: Diabetes increases a woman's risk of CAD fivefold and of MI twofold, not 10-fold, over nondiabetics. The other statements are all true. 4. All of the following are true regarding the use of hormone replacement therapy for prevention of coronary artery disease except: a. Estrogen therapy increases the risk of breast and endometrial cancer. P.742

b. Hormone replacement therapy is useful for secondary prevention of CAD in women. c. The HERS trial showed no difference in nonfatal MI or coronary death in postmenopausal women with prior MI or revascularization or angiographic CAD who had hormone replacement therapy. d. Hormone replacement therapy may increase risk of CAD in healthy postmenopausal women. View Answer Answer is b: Hormone replacement therapy has no role in either primary or secondary prevention of coronary artery disease. It may, in fact, increase the risk of CAD in healthy postmenopausal women. HRT has certain noncardiac risks associated with it, including increased risk of breast cancer, endometrial cancer, and gallbladder disease, as well as increased risk of DVT and pulmonary embolism. 5. The use of thrombolytics in women who present with an MI: a. Is associated with comparable infarct-related artery patency as seen in men b. Is associated with fewer bleeding complications than are seen in men c. Did not show mortality benefit as seen in men d. Is contraindicated in menstruating women or women of reproductive age View Answer Answer is a: The use of thrombolytics in women produces equivalent rates of infarct-related artery as when used in men, as well as mortality benefit. More bleeding complications were seen in women, but this is likely related to

the uniform dosing (i.e., not weight adjusted) that was used in the early studies. These bleeding complications included intracranial hemorrhages as well as groin bleeds at the site of catheterization.

57 - Preoperative Evaluation of Cardiac Patients for Noncardiac Surgery: Stress Test, Catheterization, or Just Proceed?
QUESTIONS 1. You are asked to evaluate a 55-year-old man with a history of prior myocardial infarction in preparation for an abdominal aortic aneurysm repair. A dobutamine stress echocardiogram has been ordered and shows the following.

Your recommendation to the referring physician is a. Clear the patient for surgery with beta-blocker prophylaxis. b. Stress SPECT thallium nuclear imaging. c. Coronary angiography. d. Cancellation of surgery. e. Stent grafting, in hopes of avoiding major aortic surgery. View Answer Answer is a: The dobutamine echocardiogram demonstrates (still frames at end systole) a fixed regional wall motion abnormality involving the base of the inferior wall and base of the septum consistent with a right coronary artery territory scar. The remaining left ventricular wall segments were contracting normally at rest with a resting ejection fraction of approximately 55%. Peak dobutamine stress images reveal an appropriate improvement in wall motion involving the left anterior descending and circumflex coronary territories with no change in the wall motion involving the inferior and basil septum. These results are consistent with right coronary artery territory scar and no evidence for ischemia. Based on this negative echocardiogram for ischemia, this patient with an intermediate clinical predictor, namely, prior myocardial infarction, may be considered at a relatively low perioperative risk for a cardiac event and therefore could be cleared for his procedure with beta-blocker prophylaxis. 2. A 70-year-old man with hypertension and a recently diagnosed solitary pulmonary nodule is scheduled for wedge resection. He is otherwise healthy, active, and regularly plays 18 holes of golf. His ECG reveals left ventricular hypertrophy with secondary repolarization changes consistent with a strain pattern. Your recommendation is a. Stress echocardiography for risk stratification. b. Clear the patient for surgery. c. Coronary angiography. d. Echocardiogram. e. Stress SPECT thallium imaging. View Answer Answer is b: This 70-year-old man with hypertension is scheduled for an intermediate-risk surgery. He has a history of hypertension with ECG changes consistent with left ventricular hypertrophy. Other than his age and an abnormal baseline electrocardiogram, this gentleman has no other significant clinical predictors of perioperative risk. Reportedly, he has a very good exercise capacity, which would place him in the moderate to excellent category for functional capacity based on his ability to regularly play 18 holes of golf without difficulty. Based on the absence of significant clinical predictors, as well as a good exercise capacity, this patient can be cleared for his wedge resection with a low perioperative risk of sustaining a cardiac event. P.751

3. An 80-year-old woman with hypertension and a history of congestive heart failure recently suffered a hip fracture and is in need of stabilization. She lives with family but is known to be inactive, primarily because of arthritis. Your recommendation is to do the following: a. Clear the patient for the orthopedic procedure with beta-blocker prophylaxis and careful hemodynamic monitoring. b. Coronary angiography. c. Dobutamine stress echocardiography for risk stratification. d. Echocardiogram, and if left ventricular function is normal, clear the patient for surgery. e. Exercise stress SPECT thallium. View Answer Answer is c: Because of the one reported critical predictor, namely, the prior history of congestive heart failure, as well as a suspected poor functional capacity, this patient should undergo further risk stratification using a pharmacologic stress imaging study. Stress echocardiography can be considered a good stress imaging modality in a patient with hypertensive heart disease, as the microvascular disease associated with hypertensive heart disease may result in abnormalities in coronary flow reserve that may in turn lead to a false positive result when using nuclear perfusion imaging. 4. A 78-year-old woman with a history of chronic stable angina is scheduled for cataract surgery. Your recommendation is a. Dipyridamole stress SPECT thallium imaging. b. Coronary angiography. c. Clear the patient for cataract surgery. d. Echocardiogram, and if left ventricular function is normal, clear the patient for surgery. e. Exercise stress echocardiography. View Answer Answer is c: This 78-year-old woman does have one intermediate clinical predictor of risk, namely, stable angina pectoris. However, she was noted to have good functional capacity and is undergoing a very low-risk surgical procedure, so she can be cleared for the cataract surgery with a low anticipated risk for adverse events. 5. Your patient is a 55-year-old man who is in need of a fem-pop bypass for claudication. What do you recommend for risk stratification? a. Exercise ECG b. Angiography c. Dobutamine stress echocardiography d. Dipyridamole thallium e. Clinical evaluation View Answer

Answer is e: In any patient undergoing preoperative evaluation, an assessment of clinical predictors of risk is central to initiating an appropriate workup prior to the noncardiac surgery. If this man with vascular disease were found, on clinical evaluation, to have a history of one or more intermediate or major clinical predictors of risk, or were found to be unable to exercise to a moderate level, further risk stratification with pharmacologic stress imaging would be indicated.

58 - Pharmacokinetic and Pharmacodynamic Essentials


1. Which of the following drugs is the most potent?

a. Drug A b. Drug B c. Drug C d. Drug D View Answer Answer is a: Drug potency is related the concentration that elicits 50% of the maximal response. Therefore, when comparing drug response to concentration curves, the drug that produces 50% of the maximal response at the lowest concentration is the most potent drug. In the graph above, both drugs A and B will elicit the same maximal response, therefore they have the same efficacy; however, drug A produces its effect at a lower plasma concentration. Drug C would be considered either a partial agonist or a lower-potency and -efficacy drug, whereas drug D would be defined as a receptor antagonist. 2. Which of the following is a principle of pharmacodynamics as opposed to pharmacokinetics? a. Absorption in the gastrointestinal tract b. Distribution into the central nervous system c. Metabolism of medications in the liver d. Tolerance to medication with prolonged exposure View Answer Answer is d: Tolerance to medication with prolonged exposure is a biologic issue reflecting pharmacodynamics, P.759

whereas the others reflect concepts that determine how medications are handled by the body (pharmacokinetics). 3. What two pharmacokinetic parameters determine the half-life of a medication? a. Loading dose and clearance b. Absorption and clearance c. Volume of distribution and clearance d. Absorption and volume of distribution View Answer

Answer is c: Both the volume of distribution and the clearance determine the half-life of a medication. As can be seen from the equation,

either increasing the volume of distribution or decreaseing clearance will increae the half-life of the drug. 4. RT is a 42-year-old man with atrial fibrillation after a recent aortic valve replacement. You would like to start IV procainamide in this patient. He weighs 75 kg and has normal renal function. Your target therapeutic level for procainamide is 8 g/mL. The volume of distribution of procainamide is about 2 L/kg. What is this patient's calculated loading dose? a. 1,000 mg b. 1,200 mg c. 1,400 mg d. 1,600 mg View Answer Answer is b: Based on the equation for loading dose, volume of distribution weight serum drug concentration needed, you can determine the loading dose. Here 2 L/kg 75 kg 8 mg/L (converted from g/mL) gives a loading dose of 1,200 mg. Practically speaking, we would round this to the nearest 250 mg and limit the initial load to 1,500 to 1,750 mg. 5. Therapeutic index is defined as the ratio of TD50 to ED50. a. True b. False View Answer Answer is a: This is known as the therapeutic index of a medication.

59 - Cardiovascular MedicineEssential Pharmaceuticals


1. A 54-year-old African American woman with a history of ischemic cardiomyopathy and congestive heart failure is on stable doses of lisinopril, carvedilol, furosemide, and digoxin. She was admitted to the hospital with shortness of breath, orthopnea, and lower-extremity edema. Her jugular venous pressure was 15 cm H2O, with bilateral basilar crackles on lung examination. She P.783

was 5.6 L in negative fluid balance over 48 hours of hospitalization, and her symptoms were much resolved. On the third day of hospitalization, the patient described orthostatic dizziness, with vitals showing hypotension as well as orthostasis. Which of the following is the best next step?

a. Add an ISDN/hydralazine combination to her drug regimen because she is African American and this drug combination has been shown to increase survival among black patients with heart failure on standard therapy. b. Stop lisinopril. c. Stop digoxin. d. Reduce or decrease furosemide dose. e. Discharge home with no change in medication. View Answer Answer is d: Though the addition of ISDN/hydralazine has been shown to improve survival in black patients with congestive heart failure who are on standard therapy (beta-blockers, ACE-I, digoxin, and diuretics), it is not the appropriate time to do so when the patient has decreased filling pressures. Even though the patient is orthostatic, it is not advisable to stop ACE-I, considering that it has been shown to reduce morbidity and mortality in CHF whereas furosemide has not been shown to have that effect. Digoxin does not have much effect on blood pressure. It is not advisable to discharge the patient home on her current medications (including diuretics), considering how symptomatics he is from reduced filling pressure. 2. A 64-year-old man presented to the emergency room with crushing chest pain that he had been experiencing for the last 20 minutes. Electrocardiogram (ECG) showed 3-mm ST elevation in leads V1-V3 with reciprocal ST depression in the inferior leads. The patient received aspirin and was started on a heparin drip in the emergency room and forwarded to primary angioplasty. The patient was found to have an acute occlusion of the middle of the left anterior descending artery (LAD). The patient underwent PCI with a Cypher stent (sirolimus-coated stent) to the middle LAD. How long should you recommend the patient take clopidogrel? a. The patient should take 75 mg of clopidogrel for at least 6 months. b. The patient should take 75 of clopidogrel for at least 3 months. c. The patient should take 75 mg of clopidogrel for at least 12 months. View Answer Answer is b: Though the current guidelines recommend taking 75 mg of clopidogrel for 3 months after stenting with a sirolimus-eluting stent and for 6 months after a paclitaxel-eluting stent, many physicians are recommending therapy for 1 year after drug-eluting stent placement. The recommendations may change pending new guidelines. 3. A 62-year-old man has a history of hepatitis C infection and chronic hepatitis with a liver function test 10 times the normal level. He has had an aortic valve replacement with a St Jude mechanical prosthetic valve. In addition, he has a history of heparin-induced thrombocytopenia (HITT) complicated with pulmonary embolus. In anticipation of an elective abdominal surgery, warfarin was discontinued. The patient's INR is now <2. Which if the following choices for anticoagulation would you recommend? a. Argatroban, 2 g/kg per minute b. Lepirudin, 0.4 mg/kg bolus, then 0.15 mg/kg per hour c. Heparin bolus and drip per weight normogram

d. Enoxaparin, 1 mg/kg SC every 12 hours View Answer Answer is b: Argatroban is not recommended because the patient had chronic liver disease and argatroban is cleared hepatically. Choice b, lepirudin, is cleared renally, and the patient is not known to have renal dysfunction. Heparin is contraindicated because the patient has a history of HITT and is at increased risk for thrombotic complications, particularly with prosthetic valve. Enoxaparin is not advised because there is a significant degree of cross-reactivity with unfractionated heparin with regard to HITT. 4. A 65-year-old woman has a history of dilated cardiomyopathy, an ejection fraction of 20%, and is NYHA Class III in terms of symptoms. Currently, she is on a home regimen of ramipril, long-acting metoprolol, spironolactone, and furosemide. She presented to the emergency room with shortness of breath and fourpillow orthopnea, and PND. On examination, she was tachycardic at 110 beats/min, blood pressure was 100/50 mm Hg, and there was evidence of fluid overload. The patient was admitted to the intensive care unit for pulmonary artery catheter-guided therapy. The cardiac index was 1.7 L/min/m2, PCWP was 25 mm Hg, and pulmonary artery pressure was 70/40 mm Hg. Which of the following inotropic agents would be most appropriate for this patient? a. Isoproterenol b. Dobutamine c. Milrinone d. Dopamine View Answer Answer is c: Milrinone is an inodilator (it has positive inotropic effect and vasodilator effect). It will allow improved cardiac output as well as decreasing pulmonary artery pressure without affecting the heart rate much (the patient is already tachycardiac). Dopamine, dobutamine, and isoproterenol cause tachycardia, which in this patient P.784

will increase the heart rate to even more than 110 beats/min and worsen the patient's hemodynamics. In addition, milrinone will lower pulmonary artery pressure and LVEDP in a more predictable fashion than the other agents. 5. Which of the following diuretic agents can be used in a patient with a sulfa allergy? a. Furosemide b. Toresmide c. Ethacrynic acid d. Bumetanide View Answer Answer is c: Ethacrynic acid is the only agent listed that does not have a sulfa moiety.

60 - Cardiovascular Drug Interactions


1. Amiodarone increases the levels of all of the following except: a. Digoxin b. Simvastatin c. Pravastatin d. Metoprolol View Answer Answer is c: Hepatic CYP metabolism is a minor elimination pathway for pravastatin clearance. Amiodarone can increase the levels of digoxin, simvastatin, and metoprolol. Also, amiodarone can cause bradycardia and may have additive effects with other medications that slow heart rate. 2. Which of the following induces the CYP450 3A4 enzyme? a. Rifampin b. Amiodarone P.790

c. Grapefruit juice d. Warfarin View Answer Answer is a: Rifampin is a known inducer of multiple CYP enzymes. Amiodarone and grapefruit juice are inhibitors of this system, whereas warfarin dose not inhibit or induce the CYP3A4 enzyme. Other common inducers include phenobarbital, phenytoin, and carbamazepine. 3. Which of the following additions will significantly elevate the INR in patients on a stable dose of warfarin? a. Digoxin b. Metronidazole c. Cholestyramine d. Aspirin View Answer Answer is b: Metronidazole decreases the metabolism of warfarin, leading to increases in the INR. Cholestyramine may decrease the absorption of warfarin when taken with it, and as a general rule, medications should be taken 2 hours before or after a dose of a bile acid binder. Aspirin does not change the pharmacokinetics of warfarin; however, because of the antiplatelet effects, it may increase the risk of bleeding when the two are taken concomitantly. 4. Simvastatin and lovastatin levels can be increased by all of the following medications except:

a. Verapamil b. Diltiazem c. Sotalol d. Cyclosporine View Answer Answer is c: Sotalol is cleared renally and does not affect the levels of either simvastatin or lovastatin. Verapamil, diltiazem, and cyclsporine can inhibit the CYP3A4 enzyme, increasing the levels of these two statins. However, most statins will reach increased levels when used with cyclosporine, and close monitoring is recommended. 5. Quinidine, a potent inhibitor of the CYP2D6 enzyme, can inhibit the analgesic response to which of the following: a. Morphine sulfate b. Hydromorphone c. Fentanyl d. Codeine View Answer Answer is d: Quinidine is a potent inhibitor of CYP2D6, and codeine as a prodrug needs to be converted through this enzyme to morphine for full analgesic effect. 6. Which of the following groups of drug interactions are considered pharmacodynamic? a. Warfarin and amiodarone b. Warfarin and metronidazole c. Warfarin and clopidogrel d. Warfarin and rifampin View Answer Answer is c: Pharmacodynamic interactions are those that do not change the disposition of medications in the body but may alter the pharmacologic response. In this case, the addition of clopidogrel to warfarin increases the risk of bleeding, as one is an anticoagulant and the other is a potent antiplatelet medication. The other combinations may have pharmaocokinetic interactions, in which one drug will affect the levels of the other. 7. Which of the following medications interact with sildenafil? a. Isosorbide dinitrate b. Amiodarone c. Erythromycin d. All of the above View Answer Answer is d: The answer is not as straightforward as it seems. The interaction between sildenafil and nitrates is well known and is avoided. However, sildenafil is metabolized through the CYP3A4 enzyme system, and agents

such as amiodarone and erythromycin can increase the levels of sildenafil. Caution should be used when prescribing these drugs together, and lower initial sildenafil doses should be used.

40 - Congenital Heart Disease in the Adult 41 - Essential Echocardiographic Images in Adult Congenital Heart Disease
QUESTIONS 1. A cleft mitral valve is associated with which of the following conditions? a. Secundum ASD b. Primum ASD c. Coarctation of the aorta d. Sinus venosus ASD e. Tetralogy of Fallot View Answer Answer is b: A cleft mitral valve is part of an atrioventricular canal defect, which is due to failure of the embryonic endocardial cushions to meet and partition the heart P.484

normally. A complete endocardial cushion defect has four components: primum ASD, cleft mitral valve, inlet VSD, and a widened anteroseptal tricuspid commissure. A partial AV canal defect does not have the VSD. 2. All of the following regarding bicuspid aortic valves are true except: a. May be associated with coarctation of the aorta b. Often associated with posteriorly directed jets of AI c. Commonly seen with congenitally corrected transposition of the great vessels d. May be amenable to aortic valve repair e. Most common type involves fusion of the RCC and LCC View Answer Answer is c: The most common form is fusion of the RCC and LCC, and the mechanism of AI in those patients is prolapse of the conjoined cusp. The conjoined cusp in the case of RCC and LCC fusion is anterior, and thus the AI is directed posteriorly. At least 50% of patients with coarctation of the aorta have a bicuspid valve. A bicuspid aortic valve with severe aortic insufficiency can often be surgically repaired, depending on the expertise of the surgical center. 3. A sinus venosus ASD is most often associated with which of the following? a. Coarctation of the aorta b. Marfan syndrome c. Partial anomalous pulmonary venous drainage

d. Tetralogy of Fallot View Answer Answer is c: A sinus venosus ASD is a defect located near the junction of the IVC or SVC and the RA. It is typically difficult to see by surface echocardiogram, often requiring a TEE for diagnosis. It is usually associated with drainage of the right pulmonary veins to the RA. 4. All of the following regarding Ebstein's anomaly are true except: a. A portion of the RV is atrialized. b. TV leaflets are dysplastic and adherent to the RV. c. Common important associated defects include pulmonic stenosis or atresia. d. A PFO or secundum ASD is associated in > 50% of cases. e. A common associated defect is coarctation of the aorta. View Answer Answer is e: Ebstein's anomaly of the tricuspid valve is characterized by apical displacement of the tricuspid valve into the RV. As a result, a portion of the RV becomes atrialized. TV tissue is dysplastic, with portions of the septal and inferior leaflets becoming adherent to the RV. Clinical manifestations depend on associated conditions. An important associated defect is pulmonic stenosis or atresia. Other associations include primum ASD and VSD, and congenitally corrected transposition of the great vessels. 5. Complications associated with a subaortic membrane include all of the following except: a. Aortic insufficiency b. Endocarditis-require antibiotic prophylaxis c. Left ventricular hypertrophy d. Atrial arrhythmias e. May recur postresection View Answer Answer is d: The presence of a subaortic membrane puts a patient at increased risk of endocarditis, due to the turbulent flow caused by the membrane, so antibiotic prophylaxis is recommended. In addition, the turbulent, high-velocity jets produced by the membrane damage the aortic valve over time, and patients often develop AI that requires surgery. The subaortic membrane is a fixed obstruction, which requires the left ventricle to develop high intracavitary pressures for ejection. As the LV pumps against the fixed obstruction, LVH develops (similar to what is seen with valvular AS). Subaortic membranes are known to recur occasionally postresection, although the frequency with which this occurs is unknown.

42 - Genetics in Cardiovascular Medicine


QUESTIONS

1. Match each of the following terms with the appropriate description. (1) The process of making a protein from the DNA gene sequence (2) A series of complex events in the nucleus that involves making a single-stranded RNA copy of the DNA gene sequence (3) Uracil base-pairs with adenosine (4) Portion of the gene sequence that eventually be discarded (5) Portion of the gene sequence that ultimately encodes the final protein a. DNA b. RNA c. Transcription d. Translation e. Exon f. Intron View Answer Answers: (1) d; (2) c; (3) b; (4) f; (5) e. See text for discussion. 2. What percentage of dilated cardiomyopathies are directly inherited a. 5% b. 10% c. 35% d. 50% e. 75% View Answer Answer is c The prevalence of dilated cardiomyopathy is approximately 40 to 50 per 100,000. Most cases of dilated cardiomyopathy are idiopathic. However, up to 35% of patients are thought to have a familial component to their disease. Inheritance patterns of dilated cardiomyopathy are variable and include autosomal dominant, autosomal recessive, X-linked, and mitochondrial forms of disease. 3. A 23-year-old woman taking an oral contraceptive presents with an acute pulmonary saddle embolus. A mutation in the gene of which coagulation factor is most likely to be found via genetic screening? a. Prothrombin b. Protein C P.500

c. Thrombin d. Factor VII e. Factor V

View Answer Answer is e: Mutations in of many genes that encode proteins in the coagulation pathway have been associated with hypercoaguability. The most significant mutation that increases the risk of venous thrombosis is a point mutation in the gene for factor V, which leads to an arganine-toglutamine amino acid substitution in the factor V protein. This mutation, known as factor V Leiden, results in the loss of the activated protein C cleavage site in the factor V protein. Heterozygotes with the factor V Leiden mutation have a 10-fold increase in risk for venous thrombosis, whereas homozygotes have up to an 80-fold increase in risk. It is generally regarded that oral contraceptives increase the risk of thrombotic complications in patients with inherited thrombophilia. 4. Match the following genes/protieins with their associated diseases: (1) KCNQ1 (2) Myosin-binding protein C (3) Actin (4) Dystrophin a. Hypertrophic cardiomyopathy b. Duchene muscular dystophy c. Dilated cardiomyopathy d. Long-QT syndrome View Answer Answers: (1) d; (2) a; (3) c; (4) b. See text for discussion. 5. A 35-year-old man presents with the acute onset of 10/10 tearing chest pain. Physical exam reveals a tall slender male in moderate discomfort. Closer exam reveals a high arched palate, with long thin fingers. Cardiovascular exam is notable for pectus excavatum, and a II/VI short diastolic murmer. His father died at age 38 years of unknown cause. Genetic analysis most likely will reveal mutations in which of the following genes? a. Apo E b. -Myosin heavy chain c. Dystrophin d. Fibrillin-1 e. Troponic C View Answer Answer is d: The patient presents with clinical features of Marfan syndrome and an acute aortic dissection. Marfan syndrome arises from mutations in the gene encoding a matrix glycoprotein, fibrillin-1. Many different mutations have been described that may lead to Marfan syndrome.

You might also like